Property Practice MCQS Flashcards

1
Q

Which of the following statements best describes why the parties to a property transaction will often appoint a solicitor?

A) To ensure there are sufficient funds to finance the transaction.

B) To investigate the physical condition of the land and any buildings on it

C) To identify any potential issues and offer practical solutions

D) There will be rights, obligations and restrictions on the property that will need advising on.

A

D) There will be rights, obligations and restrictions on the property that will need advising on

(Correct. In addition, property is a very valuable asset and a property transaction involves a lot of documentation)

How well did you know this?
1
Not at all
2
3
4
5
Perfectly
2
Q

Which of the following correctly describes the seller’s aims in a property transaction?

A) Ensure there are sufficient funds to finance the purchase

B) Ensure the seller has the right to sell the property

C) Identify any third party rights affecting the property

D) Identify any third party rights affecting the property

E) Ensure the property transaction is tied in with any related transaction and meets their business requirements.

A

E) Ensure the property transaction is tied in with any related transaction and meets their business requirements.

(Correct. This is one of the seller’s aims in a property transaction)

How well did you know this?
1
Not at all
2
3
4
5
Perfectly
3
Q

Which of the following correctly describes the buyer’s aims in a property transaction?

A) Identify the rights that the property enjoys.

B) Effect a legal transfer of the property to the buyer

C) Realise funds from the sale transaction and repay any mortgage on the property.

D) Ensure that the buyer has the right to buy the property.

A

A) Identify the rights that the property enjoys.

(Correct. The buyer will wish to identify the different rights that benefit and burden the property)

How well did you know this?
1
Not at all
2
3
4
5
Perfectly
4
Q

In which of the following stages in a conveyancing transaction does most of the work take place?

A) Pre-contract to exchange

B) Pre-completion to completion

C) Post-completion

A

A) Pre-contract to exchange

(Correct. The most work is pre-contract when the buyer is trying to find out as much as possible about the property before becoming contractually bound)

How well did you know this?
1
Not at all
2
3
4
5
Perfectly
5
Q

At what point in a property transaction do the parties become contractually committed to the deal?

A) Completion

B) When the TR1 is executed

C) Exchange of contracts

D) When Heads of Terms are agreed

E) Registration of the transfer deed at the Land Registry

A

C) Exchange of contracts

(Correct. At exchange of contracts, the buyer will pay a deposit (usually 10% of the purchase price), and a legally binding contract comes into existence)

How well did you know this?
1
Not at all
2
3
4
5
Perfectly
6
Q

At what stage in a property transaction is the buyer recognised as the new legal owner?

A) When the TR1 is executed

B) Registration of the buyer at the Land Registry

C) Exchange of contracts

D) Completion

E) When Heads of Terms are agreed

A

B) Registration of the buyer at the Land Registry

(Correct. Legal title to the property will not pass until the transfer deed is registered at the Land Registry)

How well did you know this?
1
Not at all
2
3
4
5
Perfectly
7
Q

At which stage in the property transaction is SDLT paid?

A) Pre-contract to exchange

B) Pre-completion to completion

C) Post completion

A

C) Post completion

(Correct. SDLT is a government tax a purchaser may have to pay depending on the price of the property. If SDLT is payable, this will be paid by the buyer post-completion)

How well did you know this?
1
Not at all
2
3
4
5
Perfectly
8
Q

At which stage in a property transaction does the buyer’s solicitor request mortgage funds from the buyer’s mortgage lender?

A) Pre-completion to completion

B) Pre-contract to exchange

C) Post completion

A

A) Pre-completion to completion

(Correct. Prior to completion the buyer’s solicitor will request mortgage funds from the lender so that it has the balance of the purchase price (less the deposit paid at exchange) in its account prior to completion taking place, the point at which the balance is transferred to the seller’s solicitor)

How well did you know this?
1
Not at all
2
3
4
5
Perfectly
9
Q

Which of the following options is not a source of information about the property for the buyer?

A) The draft contract

B) Replies to standard pre-contract enquiries

C) Official copies from the Land Registry

D) Physical survey

E) Pre-contract search results

A

A) The draft contract

(Correct. Information about the property will not come from the draft contract. The contract will document the terms agreed between the parties)

How well did you know this?
1
Not at all
2
3
4
5
Perfectly
10
Q

To which party is the principle of caveat emptor most relevant?

A) It is equally relevant to both parties

B) The buyer

C) The seller

A

B) The buyer

(Correct. The principle means that a buyer takes the property as it finds it. It cannot turn around after exchange of contracts to the seller and try pull out of the deal because there is something wrong with the property. The principle is therefore most relevant for the buyer)

How well did you know this?
1
Not at all
2
3
4
5
Perfectly
11
Q

Which of the following options best describes the principle from the case of Hardy v Griffiths?

A) There is an onus on the seller to disclose any physical defects in the property.

B) A seller does not have to disclose information about rising damp or dry rot in replies to enquiries.

C) There is no onus on the seller to disclose any physical defects, it is for the buyer to discover them.

D) A buyer can claim misrepresentation if the seller fails to disclose any information in the replies to enquires. It is not even necessary for the buyer to have read the replies.

E) A buyer can pull out of a deal before contracts are exchanged.

A

C) There is no onus on the seller to disclose any physical defects, it is for the buyer to discover them

(Correct. This demonstrates the application of the principle of ‘caveat emptor’)

How well did you know this?
1
Not at all
2
3
4
5
Perfectly
12
Q

In which of the following situations would co-ownership issues not need to be considered?

A) An individual buying a house to be held on trust for his grandchildren

B) Four friends are buying a property together

C) Two individuals buying a property from which they will run their business

D) A man is buying a property by himself with a contribution to the purchase price from his brother

E) A company is buying a property

A

E) A company is buying a property

(Correct. Only one entity is buying the property here so co-ownership issues do not need to be considered)

How well did you know this?
1
Not at all
2
3
4
5
Perfectly
13
Q

Which of these following statements applies in respect of the equitable title (ownership)?

A) You can only hold the title as joint tenants

B) You can hold the title as joint tenants or tenants in common.

C) You must be over 18 to hold the title

D) You can only hold the title as tenants in common

E) There can only be a maximum of 4 who hold the title

A

B) You can hold the title as joint tenants or tenants in common

(Correct. There is more flexibility regarding the equitable ownership. There is no limit to the number of equitable owners, a minor can hold the equitable title and the equitable owners can choose to hold the title as joint tenants or tenants in common)

How well did you know this?
1
Not at all
2
3
4
5
Perfectly
14
Q

Your client is buying a 1950s house that has been lived in continuously. Which of the following correctly explains the advice you should give your client about surveys?

A) A home buyer’s survey should be carried out because this is the most detailed survey

B) A structural survey should be carried out as this is the only survey that gives a valuation.

C) A homebuyer report would be appropriate for this kind of property.

D) A basic valuation need only be carried out because this is all a lender requires

E) A full structural survey should be carried out because this is the most detailed survey

A

C) A homebuyer report would be appropriate for this kind of property

(Correct. The basic survey should not be relied upon, and a homebuyer report will give the buyer important detail about the house)

How well did you know this?
1
Not at all
2
3
4
5
Perfectly
15
Q

You are acting for the seller of a property. Which of the following types of tax do you need to consider may be payable by your client depending upon the circumstances?

A) SDLT and VAT

B) Capital Gains Tax and VAT

C) VAT only

D) SDLT only

E) Capital Gains Tax and SDLT

A

B) Capital Gains Tax and VAT

(Correct. Capital Gains Tax (CGT) is a tax on the profit when you sell of an asset. It may therefore be payable depending upon the circumstances. VAT (value added tax) is a tax where a business charges VAT on its sales and recovers VAT on its purchases. It may be payable, you would need more information from the client)

How well did you know this?
1
Not at all
2
3
4
5
Perfectly
16
Q

Two clients are buying a property together. They are contributing in equal proportions to the purchase price but have stated they want to be able to pass their share in the property under a will should they die during the ownership of the property. Which of the following correctly explains the best advice you should give your clients?

A) The equitable title should be held by them as joints tenants because they are contributing equal proportions to the purchase price

B) The equitable title should be held by them as joints tenants because the right of survivorship will apply

C) The equitable title should be held by them as tenants in common because the right of survivorship will not then apply

D) The legal title should be held by them as tenants in common because the right of survivorship will not then apply

E) The legal title should be held by them as joints tenants because the right of survivorship will apply

A

C) The equitable title should be held by them as tenants in common because the right of survivorship will not then apply

(Correct. The clients have advised us they do not want the right of survivorship to apply, they wish to be able to leave their share of the property under a will. Therefore, they should hold the equitable title as tenants in common, which means they are seen as owning their own share in the property)

How well did you know this?
1
Not at all
2
3
4
5
Perfectly
17
Q

Which of the following statements is correct in respect of interest rates?

A) A high risk borrower is likely to be charged a higher interest rate

B) The interest rate is the cost of debt for the lender

C) The higher the interest rate the lower the monthly payments that a borrower will make

D) A low risk borrower is likely to be charged a higher interest rate

E) A high risk borrower is likely to be charged a lower interest rate

A

A) A high risk borrower is likely to be charged a higher interest rate

(Correct. When the borrower is consider a high risk, the interest rate they will be charged will be higher. This means the cost of the debt for the borrower is higher. This reflects the risk the lender is taking by loaning to the borrower)

How well did you know this?
1
Not at all
2
3
4
5
Perfectly
18
Q

A borrower is repaying monthly sums on their mortgage loan. At the end of the term they will have repaid all sums owed through these repayments. Which of the following types of mortgage loan will the borrower have?

A) A capital repayment mortgage

B) An interest only mortgage

C) A pension mortgage

D) A sharia-compliant mortgage

E) An endowment mortgage

A

A) A capital repayment mortgage

(Correct. A Capital Repayment mortgage is the basic way of repaying all mortgages. With repayment mortgages, each month you repay some of the interest you owe plus some of the money (capital) borrowed. At the end of the period the borrower will have paid back everything they owe and will own their home outright)

How well did you know this?
1
Not at all
2
3
4
5
Perfectly
19
Q

Which of the following would be classed as a disbursement for a buyer in a property transaction?

A) SDLT

B) The purchase price

C) A mortgage loan

D) Legal Fees

E) Proceeds from any related sale

A

A) SDLT

(Correct. SDLT would be classed as a disbursement, along with search fees and Land Registry fees. A buyer must have sufficient funds to pay for these in addition to the costs of the transaction)

How well did you know this?
1
Not at all
2
3
4
5
Perfectly
20
Q

You act for a buyer who is buying a house with a loan to be secured by a mortgage. You also act for the mortgage lender.

One of the conditions of the mortgage is that the buyer will meet the balance of the purchase price from their own funds, and not further borrowing.

You are due to exchange tomorrow, or the seller has indicated that they may withdraw from the transaction. The deposit has arrived today. Your accounts department tell you that the payment has been made from a company who only deal in personal loans.

How should you proceed?

A) You should notify your lender client, and let your client know when you have their response.

B) You should advise your client that you will need to disclose this to your lender client, and if your client does not give their consent to you doing this, you will need to cease acting for both buyer and lender.

C) Under the Conveyancing Protocol, you should advise the seller’s solicitor of the issue so that they understand the cause of the delay.

D) Given the urgency of the transaction, you should exchange, but raise this issue with the client before completion.

E) You should advise your client that the loan company should have made the payment to your client, and then from their account to you, so that it would appear as a cash advance.

A

B) You should advise your client that you will need to disclose this to your lender client, and if your client does not give their consent to you doing this, you will need to cease acting for both buyer and lender

(This is correct. In residential transactions it is common for the buyer’s solicitor to act for the lender because there is unlikely to be a conflict of interest between these parties.
On the facts, you have a duty to disclose (CCS 6.4) all information material to the matter which you have knowledge, which includes the fact the balance of the purchase price is being met by a company who deal in personal loans, not the buyer ie in breach of one of the conditions of the mortgage.
The duty of confidentiality (CCS 6.3) that you owe your buyer client will override the duty, which means you cannot disclose this information without the permission of the buyer.
If the buyer does not give consent, you will have to cease to act for the lender. You would be advised to stop acting for the buyer as well)

How well did you know this?
1
Not at all
2
3
4
5
Perfectly
21
Q

A solicitor is acting for a buyer of a registered freehold title in England. Part of the purchase price is to be funded by a mortgage loan.

Which of the following options describes what the buyer’s solicitor will do after completion of the purchase?

A) Pay Stamp Duty Land Tax and register the buyer as owner of the property, and the lender as a chargee, at the Land Registry.

B) Report on title to the buyer.

C) Nothing. Completion marks the end of the transaction.

D) Exchange contracts.

E) Pay off the outstanding balance of the seller’s mortgage.

A

A) Pay Stamp Duty Land Tax and register the buyer as owner of the property, and the lender as a chargee, at the Land Registry

(Correct. These are the main tasks for the buyer’s solicitor to carry out after completion)

How well did you know this?
1
Not at all
2
3
4
5
Perfectly
22
Q

You are acting for the buyer of a property. In an effort to speed up the sale, you submit searches which will involve an expense to your client. These form part of your investigation of title. At what point in the transaction would you expect to send these?​

A) After you have ensured that the draft contract is in final form.

B) After completion.

C) After receiving a draft contract and before exchange of contracts.​

D) As soon as your client indicates that they have put an offer in on the property.

E) After exchange of contracts.​

A

C) After receiving a draft contract and before exchange of contracts.

(Correct. Any issues which might arise need to be dealt with in the contract. You don’t necessarily know who the buyer is once you have received instructions from your seller client - you need the Heads of Terms. After agreeing the contract is too late - and the contract will be agreed by exchange. After completion is far too late. The best answer here is before exchange of contracts. Often it is once you have received the Heads of Terms - you will send the official copies and the standard CPSE replies if your firm has some prepared. If not, the buyer’s solicitor will send their standard set of CPSEs and enquiries)

How well did you know this?
1
Not at all
2
3
4
5
Perfectly
23
Q

You act for a client buying a house at a price of £200,000 with the help of an 80% mortgage. Searches and enquiries have been carried out, and you have agreed with the seller’s solicitor to aim for exchange by no later than the end of next week.

You receive a call from the seller’s solicitor to say that the seller has had an offer from a third party of £210,000 and that your client will need to match it, or the seller will sell to the third party.

Your client tells you that they were already overstretching themselves, and it will be impossible to do this.

How do you advise your client?

A) Your client can see if they can persuade the seller to honour the original price, but if the seller refuses, then your client will unfortunately lose the property.

B) You advise your client that as you are holding the contract, you can exercise a lien over it and refuse to return it to the seller’s solicitor until they honour the original price agreed.

C) You should ask to see proof of the third party’s offer, as a seller may only withdraw from an existing sale if a genuine offer has been received.

D) Your client should be prepared to lose the property, but at least they can recover the costs of their searches and survey from the seller.

E) You should see whether your client wishes for you to refer it to a litigation colleague to issue proceedings to force the sale to proceed

A

A) Your client can see if they can persuade the seller to honour the original price, but if the seller refuses, then your client will unfortunately lose the property

(Correct. Until exchange of contracts, either party can walk away from the deal without incurring any liability. The seller can choose to sell to another party who is offering a higher price without any liability - this is know as ‘gazumping’ in practice. There is no obligation on the seller to prove another genuine offer has been received and the buyer would not be able to recover the costs of their searches and survey from the seller)

How well did you know this?
1
Not at all
2
3
4
5
Perfectly
24
Q

You act for a client buying a house near an industrial plant. When viewing the house, your client was concerned by offensive chemical smells coming from the plant. The seller reassured your client both verbally and in a written reply to enquiries that it only happens once or twice a year when the plant carry out a particular process.

The contract incorporated the Standard Conditions of Sale (Fifth Edition).

Following completion, your client calls to say that the plant emits the smell every weekend. Correspondence with the plant operator confirms that they carry out the process weekly, and have done for the last few years. Your client’s surveyor gives the opinion that the house is worth £20,000 less than it would be if the seller’s statement had been correct.

Which of the following options best describes if your client can make a claim for damages?

A) Your client cannot make a claim against the seller because of the principle of caveat emptor

B) Your client cannot make a claim against the seller because it is a subjective matter and most of the neighbours are not offended by the smell

C) Your client may wish to a claim for damages from the seller based on misrepresentation

D) Your client cannot make a claim for damages as it has not suffered any loss in the circumstances

E) Your client may wish to claim for damages from the search provider as this should have been flagged in the desktop environmental report

A

C) Your client may wish to a claim for damages from the seller based on misrepresentation

(Correct. Although the principle ‘caveat emptor’ means the buyer takes the property as it find its and there is no onus on the seller to disclose patent incumbrances or physical defects in the property, the seller cannot mislead the buyer by answering questions dishonestly, which it has done here. The standard conditions of sale do not require the buyer the prove they were induced to reply on the conduct or statement; it is sufficient to prove that there was an error or omissions, which there has been there. The client can therefore make a claim for damages)

How well did you know this?
1
Not at all
2
3
4
5
Perfectly
25
Q

Which is the correct order of the three registers in the Official Copies?

A) 1) Proprietorship Register, 2) Property Register, 3) Charges Register.

B) 1) Charges Register, 2) Property Register, 3) Proprietorship Register.

C) 1) Property Register, 2) Proprietorship Register, 3) Charges Register.

D) 1) Property Register, 2) Charges Register, 3) Proprietorship Register.

E) 1) Charges Register, 2) Proprietorship Register, 3) Property Register.

A

C) 1) Property Register, 2) Proprietorship Register, 3) Charges Register.

How well did you know this?
1
Not at all
2
3
4
5
Perfectly
26
Q

Which of the following documents are required to deduce title to registered land?

A) Estate agent’s particulars, official copies of the register, a survey report, title plan and copies of all documents referred to in the official copies.

B) Survey report, official copies of the register and title plan.

C) Official copies of the register.

D) Official copies of the register, title plan and copies of documents referred to in the official copies where the relevant rights have not been fully extracted.

E) Title plan, official copies of the register and copies of all documents referred to in the official copies.

A

D) Official copies of the register, title plan and copies of documents referred to in the official copies where the relevant rights have not been fully extracted.

(Correct, you have understood which documents comprise the title to registered land)

How well did you know this?
1
Not at all
2
3
4
5
Perfectly
27
Q

Which one of the following entries, appearing on the official copies, is not fully extracted, so that the buyer’s solicitor would need to see a copy of the underlying document?

A) A conveyance of the land in this title dated 17 February 1900 contains restrictive covenants details of which are set out in the schedule of restrictive covenants hereto.

B) The land has the benefit of a right of way contained in a conveyance dated 27 March 1970.

C) The Transfer to the proprietor contains a covenant to observe and perform the covenants referred to in the Charges Register and of indemnity in respect thereof.

D) Restriction: Except under an order of the Registrar no disposition by the proprietor of the land is to be registered without the consent of the proprietor of the charge dated 2 June 2003 in favour of the City and County Bank Public Limited Company referred to in the Charges Register.

E) The land has the benefit of the following right contained in a conveyance dated 15 August 2000: a right of way on foot and with vehicles over the roadway shown hatched black on the plan.

A

B) The land has the benefit of a right of way contained in a conveyance dated 27 March 1970

(Correct. The buyer’s solicitor would need to see a copy of this conveyance to establish the details of the right of way eg which piece of land the right of way is over and any restrictions on its use)

How well did you know this?
1
Not at all
2
3
4
5
Perfectly
28
Q

Which ONE of the following statements about the Property Register and its possible contents is CORRECT?

A) The Property Register describes the property using the postal address and also by referring to the Title Plan.

B) The Property Register must always contain the benefit of a right of way.

C) The Property Register contains the name of the registered proprietor.

D) An extracted right is the same thing as an excluded right.

A

A) The Property Register describes the property using the postal address and also by referring to the Title Plan.

How well did you know this?
1
Not at all
2
3
4
5
Perfectly
29
Q

The property your client is considering buying has the benefit of a right of way in the Property Register and the right of way extracted on the register does not refer to maintenance obligations.

Which ONE of the following is the most accurate list of the issues you need to consider and advise on in relation to this right?

A) Adequacy, Adoption and Registration of the burden.

B) Registration of the benefit and Registration of the burden.

C) Maintenance, Adoption, Registration of the burden and Adequacy.

D) Adequacy, Adoption, Registration of the benefit and Registration of the burden.

E) Maintenance, Adequacy, Adoption and Registration of the benefit.

A

C) Maintenance, Adoption, Registration of the burden and Adequacy.

(Correct. You have understood the four issues to consider and discuss when the property has the benefit of a right of way)

How well did you know this?
1
Not at all
2
3
4
5
Perfectly
30
Q

The following entry appears in the property register of the official copies of a property your client is purchasing:

“A right of way to pass and repass day or night on foot only over the land coloured blue. Note: the land coloured blue is shown hatched black on the filed plan.”

Which ONE of the following statements in respect of this right is CORRECT?

A) As the right of way is on foot only, the local authority cannot adopt the private road to make it into a public highway.

B) The purchaser will not have to contribute towards maintenance of the right of way as there is no obligation to do so mentioned in the right.

C) The burden of the right of way should be registered in the charges register of the servient land.

D) The right of way is not extracted and so you will need to see a copy of the deed granting the right.

E) If the right of way is not adequate for your client the solutions are either to seek insurance to cover breach, or approach the person with the benefit or go to the Upper Tribunal (Lands Chamber) to have the right modified.

A

C) The burden of the right of way should be registered in the charges register of the servient land.

(Correct. If the servient land is unregistered you would put a caution against first registration over the land so that you can ensure the burden of the right of way is registered in the charges register once the servient land is registered)

How well did you know this?
1
Not at all
2
3
4
5
Perfectly
31
Q

The seller is usually the registered proprietor. Which of the following situations would be an exception to this?

A) Where the seller is the executor of a deceased person’s estate

B) Where the seller is a limited liability partnership

C) Where the seller is a company

D) Where there is more than one individual legal owner

A

A) Where the seller is the executor of a deceased person’s estate

(This is correct. The seller is usually the registered proprietor. An exception would be where the seller is the executor of a deceased person’s estate)

How well did you know this?
1
Not at all
2
3
4
5
Perfectly
32
Q

The following extract from official copies for a freehold property in London contains the entire Proprietorship Register for that property. Please read the extract and the information that follows and then answer the question below.

Proprietorship Register

Title Absolute

Proprietor(s): LUCY WATSON AND OWAIN HUTCHISON: of 1 Greenslade Avenue, London (W12 7JH).
The value as at 7 October 2001 was £375,000.
RESTRICTION: Except under an order of the Registrar no disposition by a proprietor of the land is to be registered without the consent of the proprietor of the charge dated 7 October 2001 in favour of Rydale Bank referred to in the Charges Register.
The heads of terms for the sale of the above property state that Lucy Watson is the seller. Owain Hutchison has died.

Which ONE of the following statements is the most accurate advice to a buyer client on the issue of who will sell the property?

A) Lucy Watson and Owain Hutchison held the property as tenants in common so the buyer would need to see an official copy of Owain Hutchison’s death certificate and a second trustee should be appointed to receive the purchase monies with Lucy Watson thereby overreaching any beneficial interests in the property. Both Lucy and the second trustee will be the sellers.

B) Lucy Watson can sell on her own as she and Owain Hutchison held the property as beneficial joint tenants and on Owain’s death his beneficial interest passed to Lucy under the doctrine of survivorship so the buyer just needs to see an official copy of Owain’s death certificate.

C) Lucy Watson and Rydale Bank will sell the property together as Owain’s interest in the property passed to the Bank, who have a mortgage over the property, on his death.

D) The Restriction on the Proprietorship Register indicates that the buyer must purchase the property from Rydale Bank.

A

B) Lucy Watson can sell on her own as she and Owain Hutchison held the property as beneficial joint tenants and on Owain’s death his beneficial interest passed to Lucy under the doctrine of survivorship so the buyer just needs to see an official copy of Owain’s death certificate

(Correct. Lucy and Owain were beneficial joint tenants (because there was no tenant in common restriction in the Proprietorship Register) so on Owain’s death, Lucy could sell on her own because Owain’s beneficial interest had passed to Lucy under the doctrine of survivorship. The buyer just needs to see an official copy of Owain’s death certificate)

How well did you know this?
1
Not at all
2
3
4
5
Perfectly
33
Q

Which one of these entries in the Proprietorship Register relates to co-owners of the property?

A) RESTRICTION: No disposition by a sole proprietor of the registered estate (except a trust corporation) under which capital money arises is to be registered unless authorised by an order of the court.

B) RESTRICTION: no disposition of the registered estate by the proprietor of the registered estate is to be registered without the consent signed by the proprietor for the time being of the charge dated the 17 June 2005 in favour of Redminister Building Society referred to in the charges register.

C) The Transfer to the Proprietors contains a covenant to observe and perform the covenants referred to in the Charges Register and of indemnity in respect thereof.

A

A) RESTRICTION: No disposition by a sole proprietor of the registered estate (except a trust corporation) under which capital money arises is to be registered unless authorised by an order of the court.

(Correct. This is the tenant in common Restriction)

How well did you know this?
1
Not at all
2
3
4
5
Perfectly
34
Q

If a buyer pays the purchase money to two trustees, why does the buyer not need to worry about the beneficial interests of any other deceased co-owners?

A) Because the rule of survivorship applies.

B) Because only two people can hold the legal estate.

C) Because the buyer only needs to worry about the legal estate, not any beneficial interests.

D) Because any such interests will be overreached.

A

D) Because any such interests will be overreached.

(Correct. You have understood the concept of overreaching)

How well did you know this?
1
Not at all
2
3
4
5
Perfectly
35
Q

Which of the following statements about the entries on the official copies relating to a mortgage / charge is correct?

A) If there is a mortgage / charge over the property, there will be two entries in the Charges Register and one entry in the Proprietorship Register regarding it.

B) If there is a mortgage / charge over the property, there will be one entry in the Charges Register and one entry in the Proprietorship Register regarding it.

C) If there is a mortgage / charge over the property, there will be three entries in the Charges Register regarding it.

D) If there is a mortgage / charge over the property, there will be two entries in the Charges Register and no entries in the Proprietorship Register regarding it

A

A) If there is a mortgage / charge over the property, there will be two entries in the Charges Register and one entry in the Proprietorship Register regarding it

(Correct. A mortgage / charge will have two entries in the Charges Register giving details of the mortgage / charge and also one entry in the Proprietorship Register – the lender’s Restriction)

How well did you know this?
1
Not at all
2
3
4
5
Perfectly
36
Q

The buyer will need to be certain it acquires the land free of any mortgage a seller has. Which of the following options is NOT something the buyer’s solicitor should be doing to ensure this happens?

A) Checking in enquiries that the seller will have sufficient funds to clear the mortgage​

B) Ensuring that it is a term of the contract that the mortgage will be redeemed on completion

C) Obtaining an appropriate undertaking from the seller’s solicitor to redeem the mortgage from the proceeds of sale on completion

D) Ensuring it transfers the buyer’s purchase monies to the lender on completion so that the lender can redeem its mortgage loan before then transferring the balance to the seller

A

D) Ensuring it transfers the buyer’s purchase monies to the lender on completion so that the lender can redeem its mortgage loan before then transferring the balance to the seller

(This is correct. The buyer’s solicitor will not transfer the purchase monies to the seller’s lender, it will sell them to the seller’s solicitor. The other options are all something the buyer’s solicitor should ensure it does to make sure the buyer takes the land free of the seller’s mortgage)

How well did you know this?
1
Not at all
2
3
4
5
Perfectly
37
Q

Which one of the following could appear in the Charges Register?

A) The benefit of an easement.

B) A lender’s Restriction.

C) An indemnity covenant given by the seller when it bought the property.

D) A tenant in common Restriction.

E) A lease granted out of the property.

A

E) A lease granted out of the property.

(Correct. A lease granted out of the property burdens the property and appears in the Charges Register)

How well did you know this?
1
Not at all
2
3
4
5
Perfectly
38
Q

Which of the following best describes whether a positive covenant registered in the charges register will bind the buyer of a registered freehold property?

A) The buyer will not be bound by the positive covenant unless there is also a charge restriction in the proprietorship register.

B) The buyer will not be bound by the positive covenant despite it appearing in the charges register as positive covenants do not run with the land and so cannot bind buyers of the property.

C) The buyer will be bound by the positive covenant as it is registered in the charges register of the property.

D) The buyer will be contractually bound by the positive covenant if an indemnity covenant note appears in the proprietorship register as the seller will require an indemnity covenant from the buyer.

A

D) The buyer will be contractually bound by the positive covenant if an indemnity covenant note appears in the proprietorship register as the seller will require an indemnity covenant from the buyer.

(Correct. Positive covenants do not run with the land but can be made contractually binding if there is a chain of indemnity covenants. An indemnity covenant note on the proprietorship register shows whether the seller gave an indemnity covenant and the standard conditions in the contract require the buyer to give an indemnity covenant if the seller did)

How well did you know this?
1
Not at all
2
3
4
5
Perfectly
39
Q

The Charges Register of a property contains a restrictive covenant stating that external alterations may not be made except with the consent of the Vendor, his heirs or assigns. The seller of the property added a front porch onto the property eleven years ago without obtaining consent. The buyer of the property thinks the front porch is not in keeping with the building and is removing it on completion.

Which one of the following is the most correct on these facts?

A) The seller’s solicitor should arrange an insurance policy covering the past breach of the covenant.

B) The buyer’s solicitor should arrange an insurance policy covering the past breach of the covenant.

C) No solution is needed as, since the restrictive covenant breach occurred more than ten years ago, the person owning the land with the benefit of the covenant is unable to claim for any loss.

D) The past breach is not continuing and so no solution is needed unless the person owning the land with the benefit of the covenant has claimed they suffered loss.

A

D) The past breach is not continuing and so no solution is needed unless the person owning the land with the benefit of the covenant has claimed they suffered loss.

(Correct. As the buyer is removing the porch (the alteration which breached the restrictive covenant), the past breach will not be continuing and so a solution is not needed unless the person owning the land with the benefit of the covenant has complained about the breach or claimed for loss)

How well did you know this?
1
Not at all
2
3
4
5
Perfectly
40
Q

DKT Ltd is buying a property and is concerned about covenants that burden the land. In particular DKT is concerned that its use as a textiles dyeing factory would breach a covenant which prohibits use as a factory. It has heard from the seller that they have spoken to the person owning the land with the benefit of the covenant who has indicated that they might consent to a breach of the covenant, for a price.

What is the next step you would recommend, in order to deal with this future breach?

A) Contact the person owning the land with the benefit of the covenant to request an easement to use the property as a textile dyeing factory.

B) Contact the Upper Tribunal (Lands Chamber) to get the covenant waived or removed.

C) Contact an insurance company for a quote for insurance in respect of the proposed breach.

D) Contact the person owning the land with the benefit of the covenant to request their written consent for the proposed breach.

A

D) Contact the person owning the land with the benefit of the covenant to request their written consent for the proposed breach.

(Correct. As the person owning the land with the benefit of the covenant has already been approached insurance is no longer available as an option (otherwise getting an insurance quote would have been the first solution to attempt). We are requesting permission to breach a covenant here so easements are not relevant. The Upper Tribunal (Lands Chamber) is a last resort as it is time consuming and expensive)

How well did you know this?
1
Not at all
2
3
4
5
Perfectly
41
Q

Which one of the following statements about insurance for breach of a restrictive covenant is correct?

A) The seller will pay the initial premium for the insurance policy and thereafter the buyer will pay the annual insurance premiums for the policy.

B) Insurance for a future breach of a restrictive covenant is expensive because it is difficult for the insurer to assess the risk of the breach occurring.

C) Insurance must be tried before approaching the person owning the land with the benefit of the covenant.

D) S.19(1) FSMA 2000 prevents a solicitor from arranging an insurance policy for breach of a restrictive covenant.

E) The buyer would have to arrange any insurance policy so that it was covered (ie protected) by the policy.

A

C) Insurance must be tried before approaching the person owning the land with the benefit of the covenant

(Correct. Once the person owning the land with the benefit of the covenant (PWB) is approached insurance will be unavailable since the PWB has been alerted to the breach and is now much more likely to be aware they can make a claim for loss)

How well did you know this?
1
Not at all
2
3
4
5
Perfectly
42
Q

Which of the following best sets out the remedies, in the order they should be attempted, for a future breach by the buyer of a binding positive covenant?

A) 1) Insurance against the PWB claiming loss caused by the breach, 2) Approaching the PWB for consent to breach the covenant.

B) 1) Insurance against the PWB claiming loss caused by the breach, 2) Approaching the PWB for consent to breach the covenant, 3) Going to the Upper Tribunal (Lands Chamber) to have the covenant modified or discharged.

C) 1) Seller to remedy the breach / reduce the purchase price, 2) Insurance against the PWB claiming loss caused by the breach, 3) Approaching the PWB for consent to breach the covenant, 4) Going to the Upper Tribunal (Lands Chamber) to have the covenant modified or discharged.

D) 1) Approaching the PWB for consent to breach the covenant, 2) Insurance against the PWB claiming loss caused by the breach.

E) 1) Seller to remedy the breach / reduce the purchase price, 2) Insurance against the PWB claiming loss caused by the breach, 3) Approaching the PWB for consent to breach the covenant

A

A) 1) Insurance against the PWB claiming loss caused by the breach, 2) Approaching the PWB for consent to breach the covenant.

(Correct. As the breach is a future one to be carried out by the buyer, the seller will not remedy it. The PWB should not be approached prior to an insurance quote being obtained as once the PWB is approached insurance will not be available (it will be a condition of the insurance that the PWB has not been and will not be approached). The Upper Tribunal (Lands Chamber) is not available for positive covenants (s.84 LPA 1925))

How well did you know this?
1
Not at all
2
3
4
5
Perfectly
43
Q

Which of the following statements is the most accurate description of an epitome of title?

A) An epitome is a list of title documents relating to a specific property.

B) An epitome is a set of copies of relevant title documents with a front sheet setting out details of the documents.

C) An epitome is a set of copies of the title register.

D) An epitome is a set of original relevant title documents with a front sheet setting out details of the documents.

E) An epitome is a set of copies of relevant title documents.

A

B) An epitome is a set of copies of relevant title documents with a front sheet setting out details of the documents.

How well did you know this?
1
Not at all
2
3
4
5
Perfectly
44
Q

What are the dates for compulsory first registration for 1) transfers of land for value and 2) transfers of land other than for value?

A) 1) 1 December 1990 and 2) 31 July 1990

B) 1) 1 April 1998 and 2) 1 January 1996

C) 1) 31 July 1990 and 2) 1 December 1990

D) 1) 1 December 1990 and 2) 1 April 1998

E) 1) 1 January 1996 and 2) 1 December 1990

A

D) 1) 1 December 1990 and 2) 1 April 1998

How well did you know this?
1
Not at all
2
3
4
5
Perfectly
45
Q

A seller is selling unregistered land with the following history:

- The seller bought the land in 1977 for £3,000 with a mortgage. The 1977 conveyance states that the land is sold subject to covenants in a conveyance dated 3 June 1956.

- In 1978 the seller granted a five year lease of the land for value.

- The mortgage is still in place, and the seller intends to discharge it from the proceeds of sale.

Which of the following best describes the relevant documents to include copies of in the epitome of title for the land?

A) The 1956 conveyance and the 1977 conveyance.

B) The 1956 conveyance, the 1977 conveyance, the 1978 lease and the seller’s mortgage

C) The 1956 conveyance, the 1977 conveyance, and the seller’s mortgage.

D) The 1977 conveyance, the 1978 lease and the seller’s mortgage.

A

C) The 1956 conveyance, the 1977 conveyance, and the seller’s mortgage

(Correct. Whilst the 1956 conveyance is a pre-root document, it is cross-referred to in the root of title. The 1978 lease does not need to be included as it has expired)

How well did you know this?
1
Not at all
2
3
4
5
Perfectly
46
Q

You act for a client Susan Wilson on the sale of an industrial building. Title to the property is unregistered. The deeds packet contains the following documents:

- Conveyance dated 1988 between Acton (Millinery) Ltd and Susan Wolston.

- Bundle of searches and enquiries dated 1988.

- Mortgage dated 1988 between Royal Bank of Scotland plc and Susan Wolston.

- Planning permission dated 1990 for an extension to the building.

Which one of the following best describes the documents which must be included in the epitome?

A) All of the documents listed.

B) - Conveyance dated 1988 between Acton (Millinery) Ltd and Susan Wolston.
- Mortgage dated 1988 between Royal Bank of Scotland plc and Susan Wolston.
- Planning permission dated 1990 for an extension to the building.

C) - Conveyance dated 1988 between Acton (Millinery) Ltd and Susan Wolston.
- Mortgage dated 1988 between Royal Bank of Scotland plc and Susan Wolston.
- Bundle of searches and enquiries dated 1988.

D) - Conveyance dated 1988 between Acton (Millinery) Ltd and Susan Wolston.

E) - Conveyance dated 1988 between Acton (Millinery) Ltd and Susan Wolston.
- Mortgage dated 1988 between Royal Bank of Scotland plc and Susan Wolston.

A

E) - Conveyance dated 1988 between Acton (Millinery) Ltd and Susan Wolston.
- Mortgage dated 1988 between Royal Bank of Scotland plc and Susan Wolston.

(Correct. The 1988 conveyance is the root conveyance (the conveyance to the current seller) and must be included. The 1988 mortgage is the current seller’s mortgage, entered into at the same time as the root conveyance. This must be included in the epitome, even though it will be redeemed at completion (and even if it has already been redeemed, as it post-dates the root of title))

How well did you know this?
1
Not at all
2
3
4
5
Perfectly
47
Q

A root of title conveyance containing covenants is dated 2 September 1989. The conveyance was from a company (the seller) to an individual (the buyer). Which one of the following would be correct execution of the root?

A) The seller should have executed the conveyance by two directors or one director and the company secretary signing it. The buyer should have executed the conveyance by signing with a witness, sealing and delivering it.

B) The buyer should have executed the conveyance by two directors or one director and the company secretary signing it. The seller should have executed the conveyance by signing with a witness and delivering it.

C) The seller should have affixed their company seal in the presence of two directors or one director and the company secretary who have both signed the conveyance. The buyer should have executed the conveyance by signing with a witness, sealing and delivering it.

D) The seller should have affixed their company seal in the presence of two directors or one director and the company secretary who have both signed the conveyance. The buyer should have executed the conveyance by signing with a witness and delivering it.

E) The seller should have executed the conveyance by two directors or one director and the company secretary signing it. The buyer should have executed the conveyance by signing with a witness and delivering it.

A

C) The seller should have affixed their company seal in the presence of two directors or one director and the company secretary who have both signed the conveyance. The buyer should have executed the conveyance by signing with a witness, sealing and delivering it

(Correct. These were correct execution methods for conveyances dated pre 31 July 1990)

How well did you know this?
1
Not at all
2
3
4
5
Perfectly
48
Q

What should you do if, acting for a buyer, the epitome of title contains an unstamped root of title conveyance?

A) Get the seller to arrange for the root conveyance to be stamped as soon as possible.

B) Pull out of the purchase as you cannot retrospectively stamp a conveyance.

C) Arrange for the seller to get the root conveyance stamped as soon as possible after completion.

D) Nothing as it is only a problem if one of the parties wants to rely on it in court.

A

A) Get the seller to arrange for the root conveyance to be stamped as soon as possible

(This is correct. You would need to see evidence of this before exchange and completion)

How well did you know this?
1
Not at all
2
3
4
5
Perfectly
49
Q

You are acting for the buyer of an unregistered property and the seller’s solicitor has provided you with the epitome of title to the property. The epitome contains:a 1989 conveyance and a 1977 conveyance.

The 1989 conveyance contains a PD stamp and sets out the extent of the property by reference to the 1977 conveyance.

The 1977 conveyance contains a plan showing the extent of the property and has a PD stamp and an ad valorem stamp.

Which of the following statements is correct?

A) The 1989 conveyance could be a good root of title provided it contains a certificate of value.

B) The 1989 conveyance cannot be a good root of title as it does not contain a plan showing the extent of the property.

C) The 1989 conveyance cannot be a good root of title as the property should have been registered for the first time when it was conveyed in 1989

D) The 1977 conveyance is the good root of title as it is the only conveyance which complies with all the requirements of a good root of title.

A

A) The 1989 conveyance could be a good root of title provided it contains a certificate of value

(Correct. The 1989 conveyance does not have an ad valorem stamp but provided it has a certificate of value (to go with the PD stamp) then it has been correctly stamped. The last date for compulsory first registration was 1 December 1990 and you have no information to tell you that the property was in an area that was subject to compulsory first registration before this date. Provided the 1977 conveyance (with plan) is included in the epitome, it does not matter that the description of the property in the 1989 conveyance is only by reference to the 1977 conveyance)

How well did you know this?
1
Not at all
2
3
4
5
Perfectly
50
Q

Which one of the following statements is correct regarding covenants in an unregistered title?

A) Positive covenants bind a purchaser if they are registered as a class d(ii) land charge.

B) Positive covenants can be ignored as they do not bind the land.

C) An indemnity covenant means that the original covenantor is not liable for the covenants to which the indemnity covenant relates.

D) Restrictive covenants bind a purchaser if they are registered as a class d(ii) land charge.

E) If a deed refers back to an earlier deed containing covenants, then those covenants will only be relevant if the earlier deed is provided in the epitome.

A

D) Restrictive covenants bind a purchaser if they are registered as a class d(ii) land charge

(Correct. The registration of the restrictive covenant allows it to bind a purchaser)

How well did you know this?
1
Not at all
2
3
4
5
Perfectly
51
Q

A seller has given a first mortgage over its unregistered property to A Bank PLC and a second mortgage to B Bank PLC.

Which of these would you expect to see registered at the Central Land Charges Registry?

A) Neither mortgage.

B) Neither. Both mortgages appear on the charges register.

C) The mortgage to A Bank PLC.

D) The mortgage to B Bank PLC.

E) Both mortgages.

A

D) The mortgage to B Bank PLC

(Correct. This second mortgage is a puisne mortgage which is protected by a Class C(i) land charge)

How well did you know this?
1
Not at all
2
3
4
5
Perfectly
52
Q

Assume the year is 2022. Your client is buying an unregistered property (the ‘Property’).

The root of title is a conveyance dated 1 December 1989 between Joanne Holmes (Vendor) and Edouard Matignon (Purchaser).

The root of title includes reference to the fact the Property is subject to covenants created in a conveyance dated 3 January 1955 made between Harold Branagh (Vendor) and Alison Sykes and Jonathan Sykes (Purchasers).

Which one of the following is the correct list of the CLC searches that should be carried out?

A) CLC searches against
1) Harald Branagh from 1925 – 1955,
2) Alison Sykes and Jonathan Sykes from 1955 – 1989,
3) Joanne Holmes from 1955 – 1989; and
4) Edouard Matignon 1989 - 2022

B) CLC searches against
1) Harold Branagh from 1926 – 1955,
2) Alison Sykes and Jonathan Sykes from 1955 – 1989,
3) Jo Holmes and Joanne Holmes from 1955 – 1989; and
4) Edouard Matignon 1989 - 2022

C) CLC searches against
1) Harold Branagh from 1926 – 1955,
2) Alison Sykes and Jonathan Sykes from 1955 – 1989,
3) Joanne Holmes from 1955 – 1989; and
4) Edouard Matignon 1989 - 2022.

D) CLC searches against
1) Harold Branagh from 1926 – 2022,
2) Alyson Sykes and Jonathan Sykes from 1926 – 2022,
3) Joanne Holmes from 1926 – 2022; and
4) Edward Matignon 1926 - 2022

E) CLC searches against
1) Harold Branagh from 1925 – 1955,
2) Alison Sykes and Jonathan Sykes from 1925 – 1989,
3) Joanne Holmes from 1925 – 1989; and
4) Edouard Matignon 1925 - 2022.

A

C) CLC searches against
1) Harold Branagh from 1926 – 1955,
2) Alison Sykes and Jonathan Sykes from 1955 – 1989,
3) Joanne Holmes from 1955 – 1989; and
4) Edouard Matignon 1989 - 2022.

(Correct. Those are the estate owners with the correct spelling of their names and the search periods are correct)

How well did you know this?
1
Not at all
2
3
4
5
Perfectly
53
Q

A buyer is buying an unregistered freehold property. The epitome reveals the following information: Company A sold the property in 1945 to Company B and Company B sold the property in 2003 to Company C. Company C is the current owner and seller of the property.

Which one of the following best sets out the Central Land Charges searches (Form K15) (‘CLC searches’) which the buyer’s solicitor should carry out pre-exchange?

A) CLC searches against: (1) Company A for the period 1925 – 1945; (2) Company B for the period 1945 – 2003; (3) Company C for the period 1945 – 2003.

B) CLC searches against: (1) Company A for the period 1926 – 1945; (2) Company B for the period 1945 – 2003; (3) Company C for the period 2003 – present date.

C) None. These are pre-completion searches.

D) CLC searches against: (1) Company A for the period 1926 – 1945; (2) Company B for the period 1945 – present date; (3) Company C for the period 2003 - present date.

E) CLC searches against: (1) Company A for the period 1926 –present date; (2) Company B for the period 1926 – present date; (3) Company C for the period 1926 – present date.

A

B) CLC searches against: (1) Company A for the period 1926 – 1945; (2) Company B for the period 1945 – 2003; (3) Company C for the period 2003 – present date

(Correct. CLC searches are carried out against previous owners of the property for their periods of ownership.
You would need a CLC search against Company A. As you don’t know when they bought the property, you would carry out the search from 1926, the year when the CLC register was started. However, you do know that Company A sold the property in 1945, so you only need search up to then.
The CLC search against Company B will be for the period 1945 to 2003, as you know that this is the period that they owned it.
The CLC search against Company C will be for the period of their ownership, being 2003 to the current year.
Note that the searches against companies A and B can be relied upon whenever they are dated, so if the seller’s solicitor provides them, the buyer’s solicitor need not carry out fresh searches. However, the search against Company C will need to be carried out again just before completion to confer priority)

How well did you know this?
1
Not at all
2
3
4
5
Perfectly
54
Q

A solicitor is acting for a lender which is lending 80% of the purchase price to the buyer of a registered commercial property. The buyer has agreed to the lender taking a first ranking fixed charge over the property after completion. The buyer has also agreed not to deal with or dispose of the property during the term of the loan without X Bank’s consent and for all relevant entries to be made in the Land Registry official copies.

What will the lender’s solicitor look for when examining the Land Registry official copy of the title to the property after completion to confirm the relevant entry or entries relating to X Bank have been correctly registered?

A) An entry referring to X Bank’s charge in the Charges Register.

B) An entry referring to a restriction on dealings in the Proprietorship Register and an entry referring to X Bank’s charge in the Charges Register.

C) An entry referring to a restriction on dealings in the Charges Register and an entry referring to X Bank’s charge in the Proprietorship Register.

D) An entry referring to a restriction on dealings in the Property Register and an entry referring to X Bank’s charge in the Charges Register.

E) A restriction on dealings in the Proprietorship Register.

A

B) An entry referring to a restriction on dealings in the Proprietorship Register and an entry referring to X Bank’s charge in the Charges Register

(This is correct. Charges such as mortgages are recorded in the Charges Register and any entries restricting the current proprietor’s ability to deal with the property are recorded in the Proprietorship Register)

How well did you know this?
1
Not at all
2
3
4
5
Perfectly
55
Q

You are acting for the buyer of a residential house on a plot of registered freehold land which is part of a new development so the buyer is buying part of a larger plot. The buyer has agreed to enter into the following covenants: (1) “not to use the Property for anything other than a private residence” and (2) “to erect and hereafter maintain a fence of no less than 2 metres in height along the northern and western boundaries of the Property”.

What types of covenants are these and on which of the registers at Land Registry for the title to the Property would you expect to see the two covenants registered?

A) (1) and (2) are both positive covenants and would be registered on the Charges Register.

B) (1) is a positive covenant and (2) is a restrictive covenant and both would be registered on the Proprietorship Register.

C) (1) is a restrictive covenant and (2) is a positive covenant and only (1) would be registered on the Charges Register.

D) (1) and (2) are both restrictive covenants and both would be registered on the Charges Register.

E) (1) is a restrictive covenant and (2) is a positive covenant and both would be registered on the Charges Register.

A

E) (1) is a restrictive covenant and (2) is a positive covenant and both would be registered on the Charges Register.

(Correct. Both restrictive and positive covenants are registered on the Charges register at the Land Registry. The Proprietorship Register deals with restrictions on the owner of the land’s ability to deal with it rather than restrictions over the land.
Restrictive covenants are contracts entered into between the covenantor (the owner of the burdened land) and the covenantee (the owner of the land taking the benefit) which restrict the right of the covenantor and their successors in title to freely use the burdened land eg a restrictive covenant not to use the property for anything other than a dwelling house. The covenant could be worded positively but still be a restrictive covenant eg a covenant to use the property only as a residential dwelling.
Positive covenants are contracts entered into between the covenantor (the owner of the burdened land) and the covenantee (the owner of the land taking the benefit) which require the covenantor to do something with / on / to the property e.g. to build and maintain a fence along the boundary of the property. The covenant could be worded restrictively but still be a positive covenant e.g. not to allow the boundary fence to fall into disrepair.
An aid to working out whether a covenant is restrictive or positive (regardless of whether it is worded negatively or positively) is the ‘hand in the pocket test’. If you have to ‘put your hand in your pocket’ i.e. pay out money to comply with the covenant then it is a positive covenant (one obliging you to do something))

How well did you know this?
1
Not at all
2
3
4
5
Perfectly
56
Q

You are acting for the buyers of a property. The seller is Mark Arthur. The seller’s solicitor tells you that Philip Arthur recently died. You review the official copies, which contain the following entry:

Proprietor(s): PHILIP ARTHUR and MARK ARTHUR of 35 Yewdale Road, Leeds, LS3 8QP

RESTRICTION: no disposition by a sole proprietor of the registered estate (except a trust corporation) under which capital money arises is to be registered unless authorised by an order of the court.

Which one of the following statements best explains what you need to do in respect of this entry in the Proprietorship Register?

A) You need to see a certified copy of the death certificate and then Mark Arthur can sell as a sole proprietor.

B) You do not need to do anything, the right of survivorship will apply so Mark Arthur can sell as a sole proprietor.

C) You need to ensure another trustee is appointed to pay the purchase money to so the restriction will not apply.

D) You do not need to do anything as Philip Arthur has died so the restriction is no longer relevant.

E) You do not need to do anything, this is for the seller’s solicitors to deal with.

A

C) You need to ensure another trustee is appointed to pay the purchase money to so the restriction will not apply

(This is correct. This restriction tells us Mark and Philip are beneficial tenants in common and prevents a sale by a sole owner. This means a sale by Mark alone is not permitted and would not be registered by the Land Registry. A second trustee needs to be appointed in order to comply with the restriction. This will overreach Philip’s beneficial interest, which will have passed under his estate because the right of survivorship will not have applied on his death)

How well did you know this?
1
Not at all
2
3
4
5
Perfectly
57
Q

Your firm is investigating title on behalf of a client who is buying a commercial property. The seller’s solicitor has deduced title. There is a restrictive covenant in the Charges Register of the Official Copies for the Property not to make any alterations to the property. In the CPSE Replies the seller admits that they have breached this restrictive covenant by building an extension to the rear of the property. There is no indemnity covenant in the Proprietorship Register.

Which one of the following options is the best advice for your client?

A) Restrictive covenants do not bind successors in title unless there is an indemnity covenant on the Proprietorship Register.

B) Restrictive covenants bind successors in title so the buyer should obtain restrictive covenant insurance to cover liability for breach of covenant.

C) Restrictive covenants bind successors in title so the seller should obtain restrictive covenant insurance to cover liability for breach of covenant.

D) Your client should not proceed with the purchase as the title is defective.

E) Restrictive covenants do not bind successors in title so no further action is required.

A

C) Restrictive covenants bind successors in title so the seller should obtain restrictive covenant insurance to cover liability for breach of covenant

(Correct. The burden of restrictive covenants bind successors in title as long as they are correctly registered, which is likely, as the restrictive covenant referred to is in the Charges Register of the Official Copies. As restrictive covenants run with the land and bind successors in title, they do not require an indemnity covenant in the Proprietorship Register to make them binding unlike a positive covenant.The seller has breached this restrictive covenant, and as a past breach it is the seller who is responsible for obtaining restrictive covenant insurance. While the other answer options might sound plausible, they are each incorrect. Note also that a firm of solicitors cannot arrange an insurance policy under section 19(1) FSMA 2000 unless they are authorised or exempt)

How well did you know this?
1
Not at all
2
3
4
5
Perfectly
58
Q

You act for a client company who is buying a piece of registered, undeveloped land that only has access to adopted highway via a private right of way. The burden of the right of way is registered against the registered title of the servient land. The wording of the right is as follows: “… to pass and repass with or without motorcars over the lane coloured blue …”

The lane coloured blue is just large enough to allow one car at a time to drive along it. The client intends to develop the land as an office for the client’s business with just two car parking spaces for the company directors.

Is the right likely to be sufficient for the client’s plans for the property?

A) Yes. It is legally sufficient and physically adequate.

B) No. It is not legally sufficient, but it is physically adequate.

C) Yes. It is legally sufficient and physically adequate. However, the right should additionally be checked on the CON29 replies.

D) No. It is neither legally sufficient nor physically adequate.

E) No. It is legally sufficient, but not physically adequate.

A

D) No. It is neither legally sufficient nor physically adequate

(Correct. Although the lane may be sufficient for the ultimate intended use of the property, it will not be sufficient for the construction of the office. Construction requires heavy vehicles such as dumper trucks and cement mixers. They will not be able to access the site due to both the physical and legal constraints of the right of way)

How well did you know this?
1
Not at all
2
3
4
5
Perfectly
59
Q

When is a report on title sent to a purchaser?

A) After exchange of contracts.

B) Before exchange of contracts

C) After completion.

D) Upon being instructed by the client

A

B) Before exchange of contracts

(This is correct. It is a report setting out all the matters relating to a property that the purchaser must review and make their mind up whether to buy the property. A purchaser is committed to purchase the property at exchange of contracts therefore the report must be sent out in good time before this point. It is unlikely that as soon as a solicitor has been instructed by the client that they would have had the search results and carried out the title investigation. It usually takes a few weeks for a solicitor to carry out the title investigation and then compile the report)

How well did you know this?
1
Not at all
2
3
4
5
Perfectly
60
Q

Which ONE of the following matters is typically reported on in a report on title?

A) An opinion on the value of the property.

B) Details of the inspection of the property by the conveyancer.

C) Details of the conveyancer’s terms and conditions of business.

D) Details of any rights benefiting the property.

A

D) Details of any rights benefiting the property.

(This is correct. The rights benefitting the property will be evident in the conveyancer’s title investigation of the property. The conveyancer would not have inspected the property and indeed it would make it clear as a limitation in the report that such an inspection had not taken place. The conveyancer’s terms and conditions of business would have been sent to the client upon being instructed. The report will not advise on the value of the property and whether the property is being purchased for the right price. This is something a surveyor would be able to advise on, not a solicitor)

How well did you know this?
1
Not at all
2
3
4
5
Perfectly
61
Q

A solicitor is preparing a report on title. Which of the following approaches will best help the solicitor meet its conduct requirements?

A) Giving the client only that information which would put the client off continuing with the purchase in a way that the client can understand.

B) Giving the client only that information which would put the client off continuing with the purchase, and it is for the client to question anything that they do not understand.

C) Giving the client all material information of which the solicitor has knowledge in a way that the client can understand.

D) Giving the client all material information of which the solicitor has knowledge in precise language, and it is for the client to question anything that they do not understand.

A

C) Giving the client all material information of which the solicitor has knowledge in a way that the client can understand

(Correct. A well drafted report on title will help with these requirements)

How well did you know this?
1
Not at all
2
3
4
5
Perfectly
62
Q

From where would you order any official searches from?

A) The Land Registry

B) Companies House

C) National Land Information Service

D) A channel provider, such as SearchFlow

E) The Local Authority in which the property is located

A

D) A channel provider, such as SearchFlow

(Correct. The searches themselves are provided by channel providers (who offer an ‘ordering service’), such as SearchFlow, Thames Water Property Searches and GlobalX. Requesting searches via such channel providers is a typical trainee / paralegal job)

How well did you know this?
1
Not at all
2
3
4
5
Perfectly
63
Q

Which of the following searches does NOT form part of the ‘local search’?

A) Enquiries of the Local Authority (CON 29)

B) Local Land Charges Search (LLC1)​

C) Additional Local Authority Enquiries (CON 29O)

D) Highways search

A

D) Highways search

(Correct. This is a search with the Highways Authority, rather than the Local Authority. The Local Search are enquiries with the Local Authority of the property)

How well did you know this?
1
Not at all
2
3
4
5
Perfectly
64
Q

Which of the following searches would you always order in a transaction?

A) Con29O (additional local search enquiries)

B) Waterways search

C) Company search

D) CON 29M (Coal Mining and Brine Search)

E) Drainage and Water

A

E) Drainage and Water

(Correct. This search will always be carried out. The search checks matters such as whether foul and surface water from the property drain to a public sewer, and whether the property is connected to a mains water supply)

How well did you know this?
1
Not at all
2
3
4
5
Perfectly
65
Q

Where would you expect to find out whether a public footpath crosses a property?

A) The Local Authority Search Result (CON29).

B) The Local Land Charges Search Result (LLC1).

C) The Property Register of the Official Copy.

D) Search of the Index Map Result (SIMR).

A

A) The Local Authority Search Result (CON29)

(Correct. There is a question to the local authority at enquiry 2.2 asking if there are any public rights of way (which would include footpaths) that abut or cross the property.
The Local Land Charges Search Result does not record details of footpaths, instead it records charges such as general and specific financial charges, planning charges, listed buildings charges. The Property Register of the Official Copy reveals rights benefitting the property such as private rights of way, not public footpaths. The Search of the Index Map Result reveals whether the area searched is registered or unregistered land, it does not provide any information on an any specific rights of way and indeed the Land Registry does not record public footpaths on its registers)

How well did you know this?
1
Not at all
2
3
4
5
Perfectly
66
Q

Your client is buying a house that has just had a loft conversion. Your client has asked you whether you can tell from the pre-contract papers whether the seller obtained any building regulation approval relating to the loft conversion. Where would you expect to find the answer?

A) The Local Authority Search Optional Enquiries (CON29O)

B) The Local Land Charges Search Result (LLC1).

C) The Local Authority Search Result (CON29).

A

C) The Local Authority Search Result (CON29).

(Correct. There is a question to the local authority at enquiry 1.1(f) asking if there are any building regulation approvals and completion certificates issued in respect of the property)

How well did you know this?
1
Not at all
2
3
4
5
Perfectly
67
Q

Which of the following statements is correct in relation to the Local Land Charges Search (LLC1)?

A) The LLC1 is an official search of the Register maintained by the Land Registry.

B) The LLC1 will reveal details of any application for planning permission that has been refused.

C) In every transaction the LLC1 is submitted to the local authority together with the CON29O enquiries.

D) The LLC1 reveals planning permissions and restrictions against permitted development (known as an Article 4 Direction).

E) The LLC1 result will reveal whether the property registered is common land or town or village green under the Commons Registration Act 1965 or the Commons Act 2006.

A

D) The LLC1 reveals planning permissions and restrictions against permitted development (known as an Article 4 Direction).

(Correct. The LLC1 is always carried out and the result reveals among other matters planning permissions granted (not refused) and restrictions against permitted development (an Article 4 Direction). In every transaction the LLC1 is submitted to the local authority (usually via an online search platform such as Searchflow) together with the CON29. The solicitor may raise additional enquiries on CON29O but this is not required in every transaction. It is the CON29O that will reveal whether a property is registered as common land or town or village green, not the LLC1)

How well did you know this?
1
Not at all
2
3
4
5
Perfectly
68
Q

A man is the sole owner of a freehold property. The property is currently let out as office premises. The man plans to carry out internal works to the property so that it can be let out as a single private dwelling house when the current occupant’s lease comes to an end.

Will the man require planning permission for his plans for the property?

A) No, the internal works do not constitute development and nor does the change of use because it is a change to a single private dwelling house.

B) Yes, he will require planning permission for both the internal works and the change of use because they are developments which are not permitted.

C) No, the internal works do not constitute development and nor does the change of use because the change is within the same use class.

D) Yes, although he will not require planning permission for the internal works, he will require planning permission for the change of use.

E) No, the internal works do not constitute development and nor does the change of use.

A

D) Yes, although he will not require planning permission for the internal works, he will require planning permission for the change of use

(Correct. s. 55(2)(a) TCPA 1990 states that internal works do not constitute development. Classes E(g) (offices) and C3 (dwelling house) of TCP(UC)O 1987 were the relevant use classes. Changing from one use class to another is a ‘development’ because it is a material change in use’ unless the change is one that is permitted under Part 3, Schedule 2 GPDO 2015. Under the GPDO this is not a change of use that is ‘permitted development’. It would have been a change from B1 to C3 under the pre-1 September TCP(UC)O 1987 but was also not permitted under the GPDO 2015 so does not fall within the transitional provisions)

How well did you know this?
1
Not at all
2
3
4
5
Perfectly
69
Q

What is the effect of a property being in a conservation area?

A) If a planning permission is granted the development must then be commenced within three months of the planning permission. The LPA may have made an Article 4 Direction.

B) It will be impossible to obtain planning permission for any proposed development of the property and the LPA may have made an Article 4 Direction.

C) Any planning permission granted for development is likely to be subject to more onerous conditions than if the property was outside of the conservation area and the Local Planning Authority (LPA) may have made an Article 4 Direction.

D) The LPA is under a duty to make an Article 4 Direction in respect of all property in a conservation area.

E) The LPA will make an Article 4 Direction and constructing new buildings is prohibited.

A

C) Any planning permission granted for development is likely to be subject to more onerous conditions than if the property was outside of the conservation area and the Local Planning Authority (LPA) may have made an Article 4 Direction.

(Correct. Areas are designated conversation areas because they have a special character or appearance and hence it is likely that any planning conditions (e.g. bricks to be in keeping with the surrounding buildings) will be more onerous than in areas outside of such an area. Article 4 Directions are also very common in conservation areas)

How well did you know this?
1
Not at all
2
3
4
5
Perfectly
70
Q

A company is the owner of a warehouse. The property is currently vacant. The company plans to carry out substantial internal works involving removing internal structural walls so that it can be let out as a restaurant.

What planning consents will the company require?

A) Planning permission for the material change of use and building regulation consent for the internal works.

B) Building regulation consent for the internal works.

C) Planning permission and building regulation consent for the internal works.

D) Planning permission for the change of use.

E) Planning permission for the internal works, planning permission for the material change of use and building regulation consent for the internal works.

A

A) Planning permission for the material change of use and building regulation consent for the internal works

(Correct. The building works are internal works and therefore do not constitute development (s.55(2) TCPA 1990). Planning permission for the internal works is not required. The change of use of the property from a warehouse (Class B8) to a restaurant (Class E(b)) is a ‘material change of use’ which constitutes ‘development’ and hence planning permission is required for the change of use. This change does not fall within the GPDO 2015, nor would it fall within the transitional provisions (being a change in the pre-1 September TCP(UC)O 1987 from B8 to A3). As internal works are taking place, building regulation consent is required)

How well did you know this?
1
Not at all
2
3
4
5
Perfectly
71
Q

A house was built three years ago and the owner is now selling it to your client.

Which of the following statements best describes the legislation regarding planning control?

A) If planning permission had been needed, the time limit for the local authority to bring enforcement action for the seller not obtaining planning permission is 4 years from the date of substantial completion.

B) If building regulations approval had been needed, the local authority must serve an enforcement notice for the seller having not obtained it within six months from the date of completion of the building work that is in breach.

C) The seller need not have obtained building regulations approval for the building of the house.

D) If planning permission had been needed the time limit for the local authority to bring enforcement action for the seller not obtaining planning permission is 10 years from the date of substantial completion

E) If building regulations approval had been needed, the time limit for the local authority to apply for an injunction for the seller having not obtained it is 10 years from the date of completion of the building work that is in breach.

A

A) If planning permission had been needed, the time limit for the local authority to bring enforcement action for the seller not obtaining planning permission is 4 years from the date of substantial completion.

(Correct. The correct time limit for enforcement of lack of planning permission is 4 years from the date of substantial completion (s. 171B TCPA 1990))

How well did you know this?
1
Not at all
2
3
4
5
Perfectly
72
Q

Which one of the following statements is the most accurate description of a Planning Enforcement Notice?

A) It flushes out information about potential planning breaches.

B) Non-compliance could result in a fine and the local authority carrying out the necessary work at the land owner’s expense.

C) It cannot exist independently of a Planning Contravention Notice.

D) It cannot require buildings to be demolished.

E) It invites the recipient to respond about how any breach may be satisfactorily remedied.

A

B) Non-compliance could result in a fine and the local authority carrying out the necessary work at the land owner’s expense.

How well did you know this?
1
Not at all
2
3
4
5
Perfectly
73
Q

Your client is buying a house that had a substantial kitchen extension constructed recently. The seller has confirmed that buildings regulations consent has not been obtained. Which one of the following statements is good advice to give to your client in the circumstances?

A) Advise the buyer to obtain a Regularisation Certificate before exchange of contracts.

B) Advise the buyer to obtain indemnity insurance upon completion.

C) Building Regulations consent is not required because extending a house is permitted development under GPDO 2015 provided the seller complied with the criteria laid out therein

D) Request that the buyer inspects the kitchen and to report back to you if they think the works present any problems.

E) Request that the seller obtains a Regularisation Certificate before completion.
Correct
Correct. Building Regulations is concerned with how the works take place – so building materials, insulation, health and safety, fire escapes are all matters that building regulations covers. All works whether internal or external require Building Regulations. The seller should be asked to rectify the situation and obtaining a Regularisation Certificate is retrospective consent by the local planning authority that they are happy with the construction work. If you did not obtain a Regularisation Certificate, the local planning authority could apply for an injunction at any time requiring your client to alter or remove works. This would not be fair since it is the seller who should have obtained the consent and who should pay for any works that need to be carried out in order to obtain the correct approval.

A

E) Request that the seller obtains a Regularisation Certificate before completion.

(Correct. Building Regulations is concerned with how the works take place – so building materials, insulation, health and safety, fire escapes are all matters that building regulations covers. All works whether internal or external require Building Regulations. The seller should be asked to rectify the situation and obtaining a Regularisation Certificate is retrospective consent by the local planning authority that they are happy with the construction work. If you did not obtain a Regularisation Certificate, the local planning authority could apply for an injunction at any time requiring your client to alter or remove works. This would not be fair since it is the seller who should have obtained the consent and who should pay for any works that need to be carried out in order to obtain the correct approval)

How well did you know this?
1
Not at all
2
3
4
5
Perfectly
74
Q

Which one of the following is correct regarding Commercial Property Standard Enquiries (the ‘CPSEs’)?

A) The CPSEs are raised in all residential property transactions.

B) The CPSEs will be requested by the buyer’s solicitor.

C) The CPSEs are delivered after exchange of contracts.

D) No supplemental enquiries may be raised in addition the CPSEs.

A

B) The CPSEs will be requested by the buyer’s solicitor

(Correct. The buyer will request the CPSE.1 and any relevant supplemental CPSE enquiries are raised usually by email. The seller’s solicitor will then ask their client to complete and when received back, will send on to the buyer’s solicitor)

How well did you know this?
1
Not at all
2
3
4
5
Perfectly
75
Q

Which one of the following is a requirement when using the Law Society Conveyancing Protocol (the ‘Protocol’)?

A) Raising an additional enquiry to clarify issues arising out of the buyer’s mortgage.

B) Confirm and update where necessary replies to enquiries if completed more than two months earlier.

C) Additional enquiries seeking the seller’s opinion may be raised.

D) Submission to the buyer’s solicitor of Property Information Form (TA6) and CPSE.2.

E) Additional enquiries relating to any issues may be raised.

A

B) Confirm and update where necessary replies to enquiries if completed more than two months earlier

(Correct. This is from paragraph 15 of the Protocol)

How well did you know this?
1
Not at all
2
3
4
5
Perfectly
76
Q

A company is selling a piece of land. In preparing replies to enquiries, the company director states that the company “is not aware of any disputes with neighbouring properties”. The company director can find no disputes in her file for the property. However, the site manager of the land did have a significant verbal dispute (not written down) with the neighbouring land, which he did not bring to the attention of the director.

After completion, the buyer finds out about the dispute. Is the seller protected against a claim by the wording of the reply?

A) No, because the seller should not give a reply without being certain as to its truth and the wording “is not aware” has no effect.

B) Yes, because the seller has checked her records.

C) No, because the seller can only rely on written evidence to provide replies to enquiries.

D) No, the director should have spoken to the site manager.
Correct
Correct. The seller should make reasonable efforts to check records AND speak to appropriate people in the organisation

A

D) No, the director should have spoken to the site manager.

(Correct. The seller should make reasonable efforts to check records AND speak to appropriate people in the organisation)

How well did you know this?
1
Not at all
2
3
4
5
Perfectly
77
Q

Which one of the following is correct regarding the Law Society’s Conveyancing Protocol (the ‘Protocol’)?

A) The Protocol can be used in residential transactions of freehold and leasehold property.

B) The Protocol can be used in commercial transactions of freehold and leasehold property.

C) The Protocol can be used in residential transactions of newly built homes.

D) The Protocol must be followed by all solicitors.

A

A) The Protocol can be used in residential transactions of freehold and leasehold property.

How well did you know this?
1
Not at all
2
3
4
5
Perfectly
78
Q

When using the Law Society’s Conveyancing Protocol, which list accurately sets out what the solicitors agree to adopt?

A) Standard Conditions of Sale (incorporated into the contract), Property Forms, Formulae for exchange, Code for Completion by Telephone.

B) Standard Conditions of Sale (incorporated into the contract), Property Forms, Formulae for exchange, Code for Completion by Post.

C) Standard Commercial Property Conditions (incorporated into the contract), Property Forms, Formulae for exchange, Code for Completion by Post.

D) Standard Conditions of Sale (incorporated into the contract), Property Forms, Formulae for completion, Code for Completion by Post.

E) Standard Conditions of Sale (incorporated into the contract), Commercial Property Standard Enquiries, Formulae for exchange, Code for Completion by Post.

A

B) Standard Conditions of Sale (incorporated into the contract), Property Forms, Formulae for exchange, Code for Completion by Post.

How well did you know this?
1
Not at all
2
3
4
5
Perfectly
79
Q

Which one of the following actions would constitute compliance with the Law Society’s Conveyancing Protocol (‘the Protocol’)?

A) A buyer’s solicitor sending two pages of standard enquiries, as well as the Property Forms to the seller’s solicitor.

B) A seller’s solicitor sending to the buyer’s solicitor official copies of the register that are eight months old.

C) A buyer’s solicitor additional pre-contract enquiry as follows: “Has the property ever suffered from an infestation of rodents?’

D) A CQS solicitor not following the Protocol without any justification.

A

C) A buyer’s solicitor additional pre-contract enquiry as follows: “Has the property ever suffered from an infestation of rodents?’

(Correct. This is an enquiry of fact, not opinion and the seller should give a reply)

How well did you know this?
1
Not at all
2
3
4
5
Perfectly
80
Q

You act for a client who intends to buy a registered freehold in the countryside. The client has asked you to look in the pre-contract papers to see whether a public footpath crosses the property.

Where would you normally expect to find the answer?

A) Search of the Index Map Result.

B) The Commons Registration Search Result.

C) The Local Land Charges Search Result.

D) The Local Authority Search Result.

E) The Property Register of the Official Copy.

A

D) The Local Authority Search Result

(Correct. There is a question to the local authority at enquiry 2.2 asking if there are any public right of ways (which would include footpaths) that abut or cross the property. While the other answer options might sound plausible, they are each incorrect. The Local Land Charges Search Result does not record details of footpaths, instead it records charges such as general and specific financial charges, planning charges, listed buildings charges, light obstruction notices. The Property Register of the Official Copy reveals rights benefiting the property such as private rights of way, not public footpaths. The Search of the Index Map Result reveals whether the area searched is registered or unregistered land, it does not detail any specific rights of way and indeed the Land Registry does not record public footpaths on its registers. The Commons Registration Search result (this is an optional enquiry in the Local Authority Search) will reveal whether the property is registered common land or town or village green under the Commons Registration Act 1965 or the Commons Act 2006)

How well did you know this?
1
Not at all
2
3
4
5
Perfectly
81
Q

Imagine last year Proporation Limited built a patio and porch along the full width of the rear of the Property and did not obtain the requisite planning permission. Which of the following statements best sets out NHL’s potential liability if it buys the Property without planning permission being in place?​

A) The local authority can serve a stop notice on NHL requiring it to stop the ongoing breach and demolish the patio and porch.

B) The local authority can serve an enforcement notice on NHL requiring it to pay a fine of up to £20,000.

C) The local authority has four years from the date of substantial completion of the patio and porch to take enforcement action against NHL.

D) NHL will have no liability as it did not carry out the works.

E) The local authority can serve a breach of condition notice against NHL because it is a condition of lawful development that planning permission is obtained before development takes place.

A

C) The local authority has four years from the date of substantial completion of the patio and porch to take enforcement action against NHL.

How well did you know this?
1
Not at all
2
3
4
5
Perfectly
82
Q

You act for a landlord client who has received a planning enforcement notice from the local authority. The landlord owns a commercial unit (‘Premises’) and five years ago, finished converting the Premises from a storage centre to a wine bar. The enforcement notice claims the landlord’s conversion works were carried out without planning permission and the material change of use wasn’t authorised under permitted development rights.

Which of the following best describes the advice you would give to the landlord regarding the enforcement periods for the two alleged breaches of planning law?

A) The local authority can enforce against the landlord for carrying out works without planning permission because the works were carried out to effect the change of use and the time limit for changing use without authorisation is still running.

B) The local authority is unable to enforce against the landlord for either breach because they have allowed the wine bar to trade for the past five years without issuing a complaint.

C) The local authority is unable to enforce against the landlord for lack of planning permission for both the works and material change of use because service of the enforcement notice is defective – the notice should have been sent to the tenant of the Premises instead of to the landlord.

D) The local authority is out of time to enforce against the landlord for the lack of planning permission for the building work but they are within the time limit to enforce for the unauthorised change of use.

E) The local authority is out of time to enforce against the landlord for the lack of planning permission for both the works and change of use because more than four years have passed since the works and change of use were completed.

A

D) The local authority is out of time to enforce against the landlord for the lack of planning permission for the building work but they are within the time limit to enforce for the unauthorised change of use

(Correct. This answer reflects the enforcement periods for lack of planning permission for works (four years) compared to lack of planning permission for a material change of use (ten years))

How well did you know this?
1
Not at all
2
3
4
5
Perfectly
83
Q

You are acting for the purchaser of a registered commercial freehold property and in order to complete the report on title for the client you need to locate information from the searches about the following issues:

• Whether the property is a listed building

• Whether a contaminated land notice has been served.

• Building regulations previously granted.

• Flooding risk

• Whether the boundary fence needs repair.

Which of the following options best describes the searches and/or enquiries you would use to obtain the information needed?

A) CON29 search; environmental desktop search and Commercial Property Standard Enquiries.

B) CON29 search and waterways search.

C) LLC1 search; CON29 search and environmental desktop search.

D) LLC1 search; CON29 search; environmental desktop search and Commercial Property Standard Enquiries.

E) LLC1 search; CON29 search; waterways search and Commercial Property Standard Enquiries.

A

D) LLC1 search; CON29 search; environmental desktop search and Commercial Property Standard Enquiries.

(This is correct. Whether the property is listed will appear in the LLC1. Contaminated land notices and building regulations approval are only detailed in the CON29. Flooding risk appears in the environmental desktop search and the seller will confirm in the CPSEs whether the fence needs repairing)

How well did you know this?
1
Not at all
2
3
4
5
Perfectly
84
Q

You have recently been instructed to act for a buyer on the purchase of an industrial unit. The seller built a large extension to the rear of the unit in September 2008. The seller’s solicitor has sent you the replies to CPSEs and you note that the seller did not obtain building regulations approval for the extension.

Is this a problem for the buyer?

A) No as the building works were carried out over ten years ago and therefore are outside the local authority’s enforcement period re: the lack of building regulations approval.

B) No as the seller was responsible for the building works in 2008 it will not become a problem for the buyer.

C) Yes as you would need to check the building regulations approval conditions as if breached there is a ten year enforcement period by the local authority starting from the date of the breach of condition.

D) Yes as there is no time limit for enforcement re: the lack of building regulations approval by the local authority.

E) No as the building works were carried out over four years ago and therefore are outside the local authority’s enforcement period re: the lack of building regulations approval.

A

D) Yes as there is no time limit for enforcement re: the lack of building regulations approval by the local authority

(Correct. The local authority can apply to court to enforce the building regulations by injunction at any time. While the other answer options might sound plausible, they are each incorrect.
Breach of conditions apply to planning permission only.
The four year enforcement period related to planning permissions.
The ten year enforcement period related to planning permissions.
The liability for lack of building regulations approval will pass to the buyer on completion)

How well did you know this?
1
Not at all
2
3
4
5
Perfectly
85
Q

A planning officer for a local authority has become aware of a potential planning issue. Five years ago, a commercial property within the local area has been redeveloped into a single residential property without obtaining any planning permissions.

Which of the following options should the planning officer take for this breach of planning control?

A) Serve an Enforcement Notice on the proprietor as the enforcement period for this change of use is unlimited.

B) There is nothing he can do as the planning contravention is outside the enforcement period from the date of the change of use.

C) Apply to court for an injunction to restore the property to its lawful use.

D) Serve an Enforcement Notice on the proprietor as the planning contravention is within the enforcement period from the date of the change of use.

E) Nothing as the change of use is permitted as general development

A

B) There is nothing he can do as the planning contravention is outside the enforcement period from the date of the change of use

(Correct. This change of use of a building to a single residential property has a four year enforcement period which has now expired. While the other options appear plausible they are not. This change of use of a building to a single residential property has a four year enforcement period which has now expired. The enforcement period for change of use is not unlimited. An injunction applies mainly to lack of building regulations. This is not a permitted development)

How well did you know this?
1
Not at all
2
3
4
5
Perfectly
86
Q

Which of the following statements correctly describes the purpose of a certificate of title?

A) The certificate is a guarantee of a clear and good title to a property.

B) It confirms that the security over the property is valid and enforceable.

C) It sets out the risks and issues relating to the property and provides advice and opinions on those risks/issues.

D) It confirms to the recipient matters relating to the property through a series of statements of facts and disclosures

E) It sets out the terms of the security deed that the recipient is entering into.

A

D) It confirms to the recipient matters relating to the property through a series of statements of facts and disclosures

87
Q

A company is buying a registered freehold property in England with funding from a corporate lending bank secured on the property with a first legal charge. The company’s solicitor is required to submit to the lender’s solicitor a CLLS Certificate of Title (COT) as a condition precedent to receiving the funding.

When should the company’s solicitor receive confirmation from the lender’s solicitor that the draft COT is approved?

A) A week before completion.

B) A couple of hours before completion.

C) Before exchange of contracts.

D) At completion.

A

C) Before exchange of contracts

(Correct. A buyer’s solicitor should not exchange contracts unless the source of funds is confirmed. Exchanging without confirmation means the lender could have issues with the property and then not want to lend, leaving the buyer contracted to buy without access to the main source of funds. There would be financial penalties under the contract if the buyer did not complete)

88
Q

Which one of the following correctly describes section 2 of the Law of Property (Miscellaneous Provisions) Act 1989?

A) Implies certain covenants or obligations on the part of the seller.

B) Contracts for the sale of land must be in writing and all the relevant terms must be incorporated into the contract.

C) The interest of a person in actual occupation will bind a purchase of the property.

A

B) Contracts for the sale of land must be in writing and all the relevant terms must be incorporated into the contract

(Correct. This is indeed two of the requirements for a valid land contract. The contract will also need to be signed by, or on half of the parties to the contract)

89
Q

Which one of the following will be expressly incorporated into a contract for the sale of land?

A) Law Society Conveyancing Protocol

B) Solicitor undertakings

C) Standard Conditions (either the Standards Conditions of Sale or the Standard Commercial Property Conditions)

A

C) Standard Conditions (either the Standards Conditions of Sale or the Standard Commercial Property Conditions)

(Correct. The standard conditions will always be incorporated into a contract for the sale of land. They contain conditions that are common to all transactions and it is far easier to incorporate these by reference rather than write them all out again in the contract)

90
Q

Your client is selling his home and you are about to start drafting the contract. Which ONE of the following contracts would you choose as a precedent?

A) A tailor made contract for sale incorporating the standard conditions of sale only.

B) A standard form contract for sale incorporating the standard commercial property conditions only.

C) A tailor made contract for sale incorporating the standard commercial property conditions and special conditions

D) A standard form contract for sale incorporating the standard conditions of sale and pre-printed special conditions.

A

D) A standard form contract for sale incorporating the standard conditions of sale and pre-printed special conditions.

(Correct. It is usual for the solicitor to prepare the standard form contract for sale incorporating the standard conditions for sale for residential transactions and the pre-printed special conditions will be at the back page of the standard form contract. A tailor made contract is used mostly in commercial property transactions which this isn’t)

91
Q

Which of the following statements correctly describes the role of special conditions in a property contract?

A) Special conditions are incorporated into both residential and commercial transactions though there are different conditions depending upon the nature of the transaction. Both conditions are updated from time to time to take account any changes in law.

B) Special conditions may be used in residential property contracts only to add to or amend the standard conditions

C) Special conditions may be used in both residential and commercial property contracts to add to or amend the standard conditions

D) Special conditions may be used in commercial property contracts only to add to or amend the standard conditions

A

C) Special conditions may be used in both residential and commercial property contracts to add to or amend the standard conditions

(This is correct. The Standard Conditions of Sale may be amended, excluded or supplemented by ‘special conditions’. This applies in respect of both residential and commercial property transactions. Though, in respect of residential transactions, if the seller’s solicitor is following the Law Society Conveyancing Protocol, special conditions may only be added if ‘absolutely necessary for the purposes of the transaction’)

92
Q

If, immediately following exchange of contracts, the seller wishes to use the deposit payable by the buyer, the seller’s solicitor must ensure that the contract provides for the deposit to be held in which of the following capacities?

A) By the seller’s solicitor as agent for the buyer.

B) By the seller’s solicitor as agent for the seller’s estate agent.

C) By the seller’s solicitor as agent for the seller.

D) By the seller’s solicitor as stakeholder.

A

C) By the seller’s solicitor as agent for the seller

(Correct. Holding the deposit as agent for the seller means the money belongs to the seller and the seller is free to use it from exchange of contracts. If the deposit was held as stakeholder this means that the seller’s solicitor is custodian of the deposit for both parties: the deposit may not be released to the seller until completion)

93
Q

Time is usually of the essence under both the SCS and the SCPC.

Is this statement true or false?

A) True

B) False

A

B) False

(Correct. Both SCS and SCPC state that ‘time is NOT of the essence until a notice to complete is served’. This means that if a party fails to complete, the non-defaulting party cannot not yet walk away from the transaction)

94
Q

Which of the following will sell with limited title guarantee?

A) A seller who is a trustee

B) A seller who is the owner and occupier.

C) A seller who is a mortgagee in possession

D) A seller who is the owner but has fully let the property out to a tenant.

A

A) A seller who is a trustee

(Correct. A trustee would usually give only a limited title guarantee)

95
Q

Your client is buying a house from a man who is an owner and an occupier of the house. Your review of the official copies reveals the registered proprietors as the man and woman who hold the property as tenants in common. You have been informed that the woman has died. Which ONE of the following would you include in the contract?

A) The man is to transfer the property with limited title guarantee

B) The man is to transfer the property with full title guarantee and agrees to appoint a second trustee who will give limited title guarantee in the transfer deed.

C) The man is to transfer the property with limited title guarantee in the transfer deed and appoint a second trustee who will give full title guarantee in the transfer deed.

D) The man is to transfer the property with full title guarantee.

A

B) The man is to transfer the property with full title guarantee and agrees to appoint a second trustee who will give limited title guarantee in the transfer deed

(Correct. A buyer from a sole surviving tenant in common would be expecting a title guarantee from both the surviving tenant in common AND from the second trustee appointed to overreach any beneficial interest in the property. However, the second trustee is often not appointed until completion. The sole surviving tenant in common will be named as the ‘Seller’ in the contract and will agree (by way of a special condition) to appoint the second trustee on completion. A trustee will usually agree only to give limited title guarantee as it has limited knowledge of the property)

96
Q

You are acting for the buyer of a residential property. In the replies to enquiries, the seller disclosed that their adult child occupiers the property with them.

Which pre-written special condition would you expect to be included in the contract in light of this?

A) The property is sold with vacant possession - this should be included to include the details of the adult son in occupation

B) This cannot be covered by a pre-written special condition. Rather, a tailor made special condition would need to be drafted

C) Title guarantee - this should be included so that the title guarantee given by the seller is varied from full to limited

D) Occupier consent’s - the adult son should sign the contract to confirm they will vacate on or before completion

A

D) Occupier consent’s - the adult son should sign the contract to confirm they will vacate on or before completion

(This is correct. The seller’s solicitor would need to obtain the signature of the adult child, who would need to be advised to seek independent legal advice)

97
Q

A buyer and seller have agreed that the washing machine and dryer (not integrated)should be included in the sale of a property.

How should the buyer ensure this is documented in the contract?

A) The buyer should ensure pre-written special condition 3(a) is included and attach a list of any other fittings/chattels that are agreed the buyer will purchase

B) The buyer does not need to do anything as all fixtures will pass automatically to the buyer on completion of the sale

C) The buyer should ensure a tailor made special condition is drafted and included in the contract documenting the fittings/chattels the buyer would like including in the sale.

D) The buyer does not need to do anything as the standard conditions of sale include a condition that all fittings /chattels will pass automatically to the buyer on completion of the sale

E) The buyer does not need to do anything as all fittings/chattels will pass automatically to the buyer on completion of the sale

A

A) The buyer should ensure pre-written special condition 3(a) is included and attach a list of any other fittings/chattels that are agreed the buyer will purchase

(Correct. The washing machine and dryer are not fixtures and will not therefore pass automatically to the buyer on a purchase. Fittings / chattels that the buyer would like including need to be expressly included in the contract. In a residential contract there is a pre-written special condition that allows the parties to specify any fittings / chattels (also called contents) that are to be included in the sale and any fixtures which are to be excluded)

98
Q

The draft sale contract for a commercial property incorporates the SCPC, including SCPC 2. Special Condition A1 from the SCPC is not incorporated in the draft contract.

Which one of the following most accurately sets out the implication of this drafting and the reason(s) why the contract would have been drafted this way? (Refer to the text of SCPC if you need to.)

A) VAT is chargeable on the sale of the property. The property is either a new (up to 3 years old) commercial property or an old (more than 3 years old) commercial property with an option to tax.

B) VAT is not chargeable on the sale of the property. The property is an old (more than 3 years old) commercial property with no option to tax.

C) VAT is not chargeable on the sale of the property. The property is either a new (up to 3 years old) commercial property or an old (more than 3 years old) commercial property with an option to tax.

D) VAT is chargeable on the sale of the property. The property is an old (more than 3 years old) commercial property with no option to tax.

A

A) VAT is chargeable on the sale of the property. The property is either a new (up to 3 years old) commercial property or an old (more than 3 years old) commercial property with an option to tax

(Correct. SCPC 2 is the default position which is that VAT is chargeable on the sale of the property. The purchase price should state it is exclusive of VAT. The possible reasons why the sale of a commercial property would be chargeable to VAT are that it is a new commercial property or an old commercial property where the seller has made an option to tax)

99
Q

In which one of the following situations is it most important for the purchase price in the sale contract to be expressly stated to be exclusive of VAT?

A) A residential property is being sold.

B) A new (up to 3 years old) commercial property is being sold.

C) An old (more than 3 years old) commercial property is being sold. No option to tax the property has been made.

A

B) A new (up to 3 years old) commercial property is being sold

(Correct. VAT is chargeable on the sale of a new (up to 3 years old) commercial property but the default position is that the purchase price is deemed to be VAT inclusive unless the contract states otherwise. If the purchase price was not expressly stated to be exclusive of VAT it will be deemed to be VAT inclusive. The seller will have to account for the VAT element out of the purchase price to HMRC and will be left only with the remainder)

100
Q

A five year old commercial property is being sold. The seller has not opted to tax. The seller is drafting the contract, which includes the SCPC.

Which one of the following correctly sets out the contractual VAT provision(s) required and the reason why? (Refer to the text of SCPC if you need to.)

A) SCPC 2 is incorporated in the contract and the purchase price should state it is exclusive of VAT because VAT is chargeable on the sale.

B) SCPC 2 is incorporated in the contract because VAT is not chargeable on the sale.

C) SCPC special condition A1 should be expressly incorporated because VAT is not chargeable on the sale.

D) SCPC special condition A1 should be expressly incorporated and the purchase price should state it is exclusive of VAT because VAT is chargeable on the sale.

A

C) SCPC special condition A1 should be expressly incorporated because VAT is not chargeable on the sale

(Correct. The sale of an old commercial property where the seller has not opted to tax is not chargeable to VAT. SCPC special condition A1 is the relevant provision to incorporate to reflect the position that VAT is not payable in the contract)

101
Q

Which one of the following steps is taken by the buyer’s solicitor pre-exchange of contracts where the buyer is purchasing with the aid of a mortgage?

A) Draft a certificate of title and send it to the lender.

B) Preparing engrossments of the final version of the contract.

C) Replying to enquiries.

D) Obtaining a redemption figure from the seller’s lender.

A

A) Draft a certificate of title and send it to the lender

(Correct. If the purchase is being funded with the aid of a mortgage the solicitor must draft a certificate of title and send it to the lender. If the lender is satisfied with the content of the certificate then the loan will be available to the buyer on the day of completion. There are different certificates of title for commercial and residential transactions. However in both cases it will set out the same type of information about the property as a report on title (ie that the title to the property is good and marketable and acceptable security for the lender))

102
Q

What is the most common form of exchange of contracts?

A) By post, whereby the buyer’s solicitor sends his client’s part of the contract and the deposit cheque to the seller’s solicitor.

B) By telephone, whereby each solicitor has their client’s part of the contract and they agree to exchange during a telephone call.

C) In person, whereby the buyer’s solicitor attends the seller’s solicitor’s office.

A

B) By telephone, whereby each solicitor has their client’s part of the contract and they agree to exchange during a telephone call

(Correct. This is the most common method of exchanging contracts)

103
Q

What undertakings does a seller’s solicitor give when using Law Society Formula B to exchange contracts?

A) To send a client account cheque for the deposit to the buyer’s solicitor on that day.

B) To discharge the seller’s mortgage.

C) To send the seller’s part of the contract to the buyer’s solicitor on that day.

D) To send the seller’s part of the contract to the buyer’s solicitor on receipt of the buyer’s part of the contract.

A

C) To send the seller’s part of the contract to the buyer’s solicitor on that day

(Correct. You spotted the deposit is sent by the buyer’s solicitor to the seller’s solicitor and not the other way round)

104
Q

You act for the buyer of a freehold property registered with Title Absolute. The contract for the sale of the property says the seller is going to sell with limited title guarantee. You explain to your client that it is usual for this type of seller to offer a full title guarantee. Your client asks you what it means for them to getting limited title guarantee. What would your advice to the client be?​

A) Selling with limited title guarantee means the seller is selling the property free from all charges and incumbrances (monetary or otherwise) and free from all rights exercisable by third parties, including those the seller does not and could not reasonably know about. ​

B) Selling with limited title guarantee means the seller is selling the property free from all charges and incumbrances (monetary or otherwise) and free from all rights exercisable by third parties, other than those the seller does not and could not reasonably know about. ​

C) The client does not need to be concerned about obtaining limited title guarantee as the property has been registered with Title Absolute.​

D) Selling with limited title guarantee means the seller covenants that the seller is not aware of any rights having been granted over the property during the period of the seller’s ownership, but the seller doesn’t make any covenant about rights before that.

E) Selling with limited title guarantee means the guarantee is limited in time to the period for which the buyer owns the property.​

A

D) Selling with limited title guarantee means the seller covenants that the seller is not aware of any rights having been granted over the property during the period of the seller’s ownership, but the seller doesn’t make any covenant about rights before that

(Correct. The key difference of limited title guarantee from full title guarantee is that the seller is only guaranteeing what has occurred at the property during the seller’s ownership. Note that if the seller inherited or was given the property, then this period extends to the last transaction for value (ie, sale))

105
Q

A solicitor has just been instructed by a client in connection with the purchase of a commercial property. The solicitor has just received the draft contract, official copies and replies to CPSEs. The buyer’s instructions are that repairs are required to the roof at the property, and that the seller has promised to carry out these repairs prior to the completion date.

Which one of the following best represents the steps that the buyer’s solicitor must now take in connection with the contract?

A) The buyer’s solicitor should draft a special condition for insertion in the contract specifying that on or before the completion date the seller must repair the roof of the property to the reasonable satisfaction of the buyer.

B) There is no need for a special condition as the verbal agreement of the seller is sufficient.

C) A special condition would not be required, as the seller is obliged to transfer the property in good repair.

D) The buyer’s solicitor should draft a special condition for insertion in the contract specifying that the seller must repair the roof.

E) The special condition is not required as the SCPC confirms that the property is sold in the state it is in at the date of the agreement between the parties.

A

A) The buyer’s solicitor should draft a special condition for insertion in the contract specifying that on or before the completion date the seller must repair the roof of the property to the reasonable satisfaction of the buyer.

(This is correct. Special conditions are required to deal with specific matters agreed between the parties. Caveat emptor would mean that in the absence of a special condition, the responsibility for the repairs would fall to the buyer. s2 Law of Property (Miscellaneous Provisions) Act 1989 states that all the terms agreed between the parties must be incorporated in the contract, so the seller’s promise to repair the roof would not be binding without being in the contract)

106
Q

A contract for sale of an industrial unit incorporates the standard commercial property conditions (‘SCPCs’) and there are no express clauses in the contract varying the standard conditions. Contracts have exchanged and the contractual completion date is next week. Yesterday the unit caught fire and has been practically destroyed.

Which one of the following statements is correct about who assumes risk under the contract from exchange of contracts?

A) The buyer bears the risk at exchange and if it does not have insurance cover it will not be obliged to complete the purchase on the contractual completion date.

B) The seller is under no obligation to insure the property; the common law position prevails, meaning that the buyer bears the risk at exchange.

C) The common law position prevails, meaning that the seller keeps the risk from exchange.

D) The seller and the buyer are both under an obligation to insure the property and must both claim on their insurance policies.

E) The seller keeps the risk in the property and the buyer does not need to complete the purchase on the contractual completion date.

A

B) The seller is under no obligation to insure the property; the common law position prevails, meaning that the buyer bears the risk at exchange

(This is correct. Since there is no variation to the SCPCs in the contract, under condition 8.1 of the SCPC, risk in the property passes to the buyer on exchange of contracts under common law (even without an express provision). The buyer should therefore be able to claim on its insurance for the damage. If for some reason the buyer does not have insurance cover, it will have to bear its own loss for the damage resulting from the fire and it will be obliged to complete the purchase on the contractual completion date)

107
Q

You are acting for a buyer client who is purchasing a property which is subject to a mortgage. You exchanged over the telephone with the seller’s solicitor in accordance with Law Society Formula B.

Which statement best describes the undertaking or combination of undertakings the seller’s solicitor would have given to you?

A) To forward the seller’s part of the contract to you and transfer the deposit to your client account.

B) To forward the seller’s part of the contract to you and to send the deposit to the lender to discharge the mortgage and to forward proof of discharge to you on receipt.

C) To forward the seller’s part of the contract to you and to send the deposit to the lender to discharge the mortgage.

D) To forward the seller’s part of the contract to you by DX this evening.

E) To forward the seller’s part of the contract to you and to keep the deposit in general client account.

A

D) To forward the seller’s part of the contract to you by DX this evening

(Correct. As you are acting for a buyer client, in accordance with Formula B you would have received an undertaking confirming that the seller’s solicitor would send the seller’s part of the contract to you by DX this evening. The outstanding mortgage will be dealt with at completion)

108
Q

You act for the seller of a newly constructed commercial property, and have been instructed to issue a draft contract.

Which of the following describes the best basis for drafting the contract?

A) You should first ascertain from replies to CPSE1 whether the seller has opted to tax the property.

B) A contract incorporating the Standard Conditions of Sale and a special condition dealing with the VAT treatment.

C) A contract incorporating the Standard Commercial Property Conditions, and including the Part 2 A2 conditions as it is a transfer of a going concern.

D) A contract incorporating the Standard Commercial Property Conditions, and including the Part 2 A1 conditions as it is not a taxable supply.

E) A contract incorporating the Standard Commercial Property Conditions, but not incorporating any of the Part 2 VAT provisions as it is a standard-rated supply.

A

E) A contract incorporating the Standard Commercial Property Conditions, but not incorporating any of the Part 2 VAT provisions as it is a standard-rated supply.

(Correct. Newly constructed commercial property is standard rated, and the Standard Commercial Property Conditions will be most appropriate for this)

109
Q

Which one of the following is the correct form of transfer for the purchase of the whole of an unregistered freehold estate?

A) TR1 signed under hand.

B) TR1 executed by deed.

C) TP1 executed by deed.

D) Conveyance signed under hand.

A

B) TR1 executed by deed

(Correct. Section 52 of the Law of Property Act 1925 states that to transfer the legal estate in land a transfer must be by deed. Although the land is unregistered, purchase will trigger compulsory first registration of the land and so the Land Registry’s Form TR1 will be used as the Transfer Deed.
A TP1 is used when only part of a registered title is being purchased.
As s 52 of the Law of Property Act 1925 states that to transfer the legal estate in land a transfer must be by deed, it cannot be signed under hand but needs the formality of being executed as a deed)

110
Q

A company is selling a property to two individuals who intend to hold the property as tenants in unequal shares. Which one of the following would be the correct execution clause(s) to insert in panel 12 of the TR1?

A) Transferor:
Executed as a deed by [name of company]
acting by [a director and its secretary] [two directors]
Signature of Director: ……………
Signature of [Director] [Secretary]: ……
Transferee:
Signed as a deed by [full name of individual]
[signature]
In the presence of:…………[signature of witness]
[print in block capitals the full name and address of the witness]
Transferee:
Signed as a deed by [full name of individual]
[signature]
In the presence of:…………[signature of witness]
[print in block capitals the full name and address of the witness]

B) Transferor:
Executed as a deed by [name of company]
acting by [a director and its secretary] [two directors]
Signature of Director: ……………
Signature of [Director] [Secretary]: ……

C) Transferor:
Signed as a deed by [full name of individual]
[signature]
In the presence of:…………[signature of witness]
[print in block capitals the full name and address of the witness]

D) Transferor:
Signed as a deed by [full name of individual]
[signature]
In the presence of:…………[signature of witness]
[print in block capitals the full name and address of the witness]
Transferor:
Signed as a deed by [full name of individual]
[signature]
In the presence of:…………[signature of witness]
[print in block capitals the full name and address of the witness]
Transferee:
Executed as a deed by [name of company]
acting by [a director and its secretary] [two directors]
Signature of Director: ……………
Signature of [Director] [Secretary]: ……

E) Transferor:
Executed as a deed by [name of company]
acting by [a director and its secretary] [two directors]
Signature of Director: ……………
Signature of [Director] [Secretary]: ……
Transferee:
Signed as a deed by [full name of individual]
[signature]
Transferee:
Signed as a deed by [full name of individual]
[signature]

A

A) Transferor:
Executed as a deed by [name of company]
acting by [a director and its secretary] [two directors]
Signature of Director: ……………
Signature of [Director] [Secretary]: ……
Transferee:
Signed as a deed by [full name of individual]
[signature]
In the presence of:…………[signature of witness]
[print in block capitals the full name and address of the witness]
Transferee:
Signed as a deed by [full name of individual]
[signature]
In the presence of:…………[signature of witness]
[print in block capitals the full name and address of the witness]

(Correct. Both the transferor (seller) and both of the transferees (buyers) must execute the TR1. The transferor must always execute the TR1 and the transferees must execute if panels 10 (a declaration of trust to be completed if there is more than one transferee) and/or 11 (additional provisions eg an indemnity covenant) of the TR1 have been completed. Here panel 10 would have been completed as there is more than one transferee. The execution clauses set out are the appropriate ones for a company transferor and individual transferees)

111
Q

A TR1 is today being drafted for the purchase of the whole of a registered title. Which one of the following best describes what should be put in Panel 3 (date) when the TR1 is drafted?

A) The date of the contract (which has been exchanged).

B) Todays’ date.

C) Nothing. Panel 3 should be left blank until completion.

D) The completion date specified in the contract (which has been exchanged).

A

C) Nothing. Panel 3 should be left blank until completion.

(Correct. The date in panel 3 is the date the TR1 is completed. It will be filled in by hand when the solicitors complete the purchase.
Panel 3 is only filled in by hand when the solicitors actually complete the purchase (when the completion monies are received). This should be on the completion date which was specified in the contract however may not be eg if completion is delayed.
The date in panel 3 is the date the TR1 is completed and not the date of the contract, which is the date of exchange)

112
Q

The registered proprietors on the official copies of a property being sold are Mr and Mrs Smith. They are joint owners and hold as tenants in common. Mr Smith has died. Mrs Smith’s solicitor is acting as the second trustee and the appointment has been made in panel 11 of the TR1. The buyer of the property is a company which changed its name a year ago.

Which one of the following best describes the documents which will need to be sent to the Land Registry post-completion with the completed TR1?

A) Mr Smith’s death certificate.

B) The separate deed of appointment of the second trustee and the change of name certificate.

C) Mr Smith’s death certificate, the separate deed of appointment of the second trustee and the change of name certificate.

D) Mr Smith’s death certificate and the change of name certificate.

E) The change of name certificate.

A

A) Mr Smith’s death certificate

(There is no separate deed of appointment to send as the second trustee was appointed in the TR1.
The buyer’s change of name certificate does not need to be sent as the Land Registry only need evidence if the name of the seller is different from the name of the registered proprietor they have on the register of title)

113
Q

Which one of the following is the correct priority period provided by an OS1 search?

A) 15 working days from the OS1 search result.

B) 30 working days from the OS1 search result.

C) 30 days from the OS1 search result.

D) 2 months from the OS1 search result.

E) 15 days from the OS1 search result.

A

B) 30 working days from the OS1 search result

(It is a common mistake to say the priority period of an OS1 is 30 days. It is actually 30 working days.
15 working days is the protection period given by a K15 Central Land Charges search.
2 months is the time period for first registration at the Land Registry (post completion of the purchase of unregistered land).)

114
Q

Two individuals are selling Unit 20, Crawford Business Park (“the Property”) to a company, which is buying the Property for cash (ie without the need for a mortgage). The Property is registered at the Land Registry. You act for the buyer. Which one of the following sets out the pre-completion searches you need to carry out?

A) OS1 search only.

B) Bankruptcy search K16 on the seller and OS1 search.

C) Company search on the buyer and OS1 search.

D) Central Land Charges search K15 on the buyer and OS1 search.

E) Bankruptcy search K16 on the seller, Company search on the buyer and OS1 search.

A

A) OS1 search only

(As this is a registered property and your client is purchasing the whole of the title, a Form OS1 search will always be required.
A bankruptcy search is not required to be undertaken against any individual seller of registered land (any bankruptcy would appear on the register of title and therefore on the OS1 search result).
As this is not unregistered land, no Central Land Charges search (K15) is required either. Central Land Charges searches K15 are only relevant for unregistered land and would be carried out against the seller not the buyer to check for adverse entries and to give a protection period.
As no buyer’s lender is involved, there are no solvency searches needed against the buyer)

115
Q

An individual seller is selling the whole of a registered freehold residential property to an individual buyer, who is buying the Property with the aid of a mortgage. You act for the buyer and their lender. Which one of the following statements is correct on these facts?

A) You need to carry out a Form K16 bankruptcy search in respect of the seller.

B) You do not need to carry out a solvency search in respect of the buyer.

C) Solvency searches are carried out at the Land Registry.

D) You need to carry out a Form K16 bankruptcy search in respect of the buyer.

A

D) You need to carry out a Form K16 bankruptcy search in respect of the buyer.

(The lender’s solicitors carry out solvency searches against the buyer. On this scenario, you act for the buyer’s lender so you must carry the appropriate solvency search against the buyer pre-completion. As the buyer is an individual, the appropriate solvency search is a K16 bankruptcy search.
If the seller is bankrupt it will appear on the OS1 search result and so no bankruptcy search is required in respect of the seller.
Companies House searches (in respect of companies) are carried out at Companies House to check solvency pre-completion. K16 bankruptcy searches or K15 CLC searches which include bankruptcy are both carried out at the Central Land Charges Department. None of the solvency searches are carried out at the Land Registry.)

116
Q

If the seller has a mortgage over the whole of a registered property being sold, which one of the following is recommended form of undertaking for the seller’s solicitor to provide in respect of the mortgage?

A) To undertake forthwith to pay the lender the money required to redeem the mortgage and to forward Form DS1 immediately to the buyer’s solicitor.

B) To undertake to discharge the mortgage and to forward Form DS1 to the buyer’s solicitor as soon as the seller’s solicitor receives it from the lender.

C) To undertake forthwith to pay the lender the money required to redeem the mortgage and to forward Form DS1 to the buyer’s solicitor as soon as the seller’s solicitor receives it from the lender.

D) To undertake to pay the completion monies to the lender and to forward Form DS1 to the buyer’s solicitor as soon as the seller’s solicitor receives it from the lender.

E) To undertake forthwith to pay the lender the money required to redeem the mortgage and to forward the receipted mortgage immediately to the buyer’s solicitor.

A

C) To undertake forthwith to pay the lender the money required to redeem the mortgage and to forward Form DS1 to the buyer’s solicitor as soon as the seller’s solicitor receives it from the lender.

(Correct.
This is the Law Society’s recommended form of seller’s solicitor’s undertaking re: the seller’s mortgage / charge.
The seller’s solicitor is not able to discharge the mortgage as only the seller’s lender can do that.
The seller’s solicitor will need to pay the money required to redeem the mortgage to the seller’s lender. This is likely to be paid out of the completion monies but it is the redemption amount and not the amount of the completion monies that must be paid over.
In registered land the seller’s solicitor will receive a Form DS1 from the lender discharging the mortgage, a receipted mortgage is used in unregistered land to show a mortgage has been discharged.
The seller’s solicitor cannot forward the receipted mortgage or DS1 until they receive it from the seller’s solicitor so they cannot undertake to send it immediately)

117
Q

Which one of the following is the most accurate statement regarding the Completion Information and Undertakings Form?

A) The Completion Information and Undertakings Form asks for details of the bank account to send the completion monies to.

B) The Law Society’s Form TA13 must be used as the Completion Information and Undertakings Form.

C):The seller completes and returns the Completion Information and Undertakings Form.

D) The Completion Information and Undertakings Form is sent by the seller’s solicitor to the buyer’s solicitor

A

A) The Completion Information and Undertakings Form asks for details of the bank account to send the completion monies to

(The Completion Information and Undertakings Form is sent by the buyer’s solicitor to the seller’s solicitor to obtain information about completion.
Form TA13 is one example of the Completion Information and Undertakings Form. Firms can use their own in house version.
The Completion Information and Undertakings Form is completed by the seller’s solicitor and signed by the seller’s solicitor. This is important as only a solicitor can give an enforceable undertaking and the Completion Information and Undertakings Form requests an undertaking in respect of the seller’s mortgage / charge)

118
Q

Which one of the following is the most accurate summary of pre-completion requisitions on title?

A) They are undertakings which the seller’s solicitor has to provide before completion.

B) They are enquiries sent to the Land Registry regarding the title being bought.

C) They ask for key information regarding completion and deal with relevant undertakings required from the seller’s solicitor.

D) They are questions on the entries in the registers of title of the property.

A

C) They ask for key information regarding completion and deal with relevant undertakings required from the seller’s solicitor

(Questions on the registers of title are asked pre-exchange in the title investigation stage.
Whilst the pre-completion requisitions on title deal with undertakings they also ask for practical information regarding completion.
Requisitions on title are questions asked of the seller’s solicitor regarding practical information for completion and dealing with relevant undertakings required from the seller’s solicitor.)

119
Q

Which one of the following is correct regarding the Law Society’s Code for Completion by Post (the ‘Code’)?

A) The Code is compulsory when completing by post.

B) The Code contains professional undertakings.

C) The Code cannot be used in commercial property purchase transactions.

D) The Code enables the buyer’s solicitors to act as agent for the seller’s solicitors.

A

B) The Code contains professional undertakings

(The Code is a voluntary procedure for postal completion.
The Code can be used for the completion of both residential and commercial property purchases.
The Code enables the seller’s solicitors to act as agent for the buyer’s solicitors. Completion occurs at the seller’s solicitor’s office as they hold the executed TR1 and receive the completion monies)

120
Q

A solicitor acts for a buyer who is about to complete, having exchanged a week previously. The contract provides for the buyer to give an indemnity covenant. The seller has a mortgage, and the buyer is buying with the benefit of a 75% mortgage. The buyer has already approved the draft SDLT return. What documents does the solicitor need the buyer to execute or sign?

A) The transfer only.

B) The transfer deed, the mortgage deed, and the discharge of the existing mortgage only.

C) The mortgage deed only.

D) The transfer deed, the mortgage deed, and written authority to complete only.

E) The transfer and mortgage deed only.

A

E) The transfer and mortgage deed only.

(Correct. The buyer will need to execute the transfer as they are giving an indemnity covenant, and the mortgage deed because they are buying with a mortgage)

121
Q

Which one of the following is a requirement of the Law Society’s Code for Completion by Post (the ‘Code’)?

A) The seller’s solicitor is to confirm to their client in writing that completion has occurred immediately after it has occurred.

B) The TR1 must be sent to the buyer’s solicitor by the end of the day of completion.

C) Written confirmation of completion must be sent to the buyer’s solicitor as soon as possible after completion and in any event by the end of the working day following completion.

D) The seller’s solicitor must send the DS1 to the Land Registry as soon as they receive it from the seller’s lender.

A

C) Written confirmation of completion must be sent to the buyer’s solicitor as soon as possible after completion and in any event by the end of the working day following completion

(Correct. The Code says that the TR1 must be sent to the buyer’s solicitor as soon as possible after completion and in any event by the end of the working day following completion.
The Code also says that as soon as possible after completion the seller’s key holder is to be notified and authorised to make the keys available to the buyer immediately and the seller’s solicitor is to confirm the date and time completion took place to the buyer’s solicitor. The Code does not put a requirement on the seller’s solicitor to notify their client as it governs the process of completion between the two solicitors. The seller’s solicitor will of course let their client know completion has occurred as soon as possible as best practice.
The Code does not place a requirement re: the DS1 on the seller’s solicitor. The seller’s solicitor will fulfil the terms of their undertaking regarding any seller’s mortgage / charge but the usual form of undertaking will be to send the DS1 to the buyer’s solicitor. It will then be the buyer’s solicitor’s post-completion task to arrange for the mortgage to be removed at the Land Registry)

122
Q

It is the contractual completion date and the buyer of a residential property, where the contract incorporates the Standard Conditions of Sale (SCS), fails to complete. The seller is ready, able and willing to complete. Which one of the following statements regarding the seller’s remedies is most accurate?

A) The seller is immediately entitled to rescind the contract and keep the buyer’s deposit because time will be of the essence as soon as the completion date has passed.

B) If the seller serves a notice to complete, the buyer fails to complete in accordance with the notice and the seller rescinds the contract, the seller is no longer entitled to contractual compensation under the SCS.

C) The seller is entitled to rescind the contract and forfeit the buyer’s deposit ten working days after the contractual completion date.

D) If the seller serves a notice to complete and the buyer fails to complete before the notice has expired, the contract will be deemed to be automatically rescinded immediately upon the expiry of the notice period.

A

B) If the seller serves a notice to complete, the buyer fails to complete in accordance with the notice and the seller rescinds the contract, the seller is no longer entitled to contractual compensation under the SCS

(The SCS compensation is for delayed completion. Completion has to actually occur for the compensation to be payable. If the contract is rescinded, completion cannot occur and there is no entitlement to contractual compensation although common law damages could be claimed for any loss suffered from the breach of contract.
Time is not of the essence under the SCS until a notice to complete is served. The notice needs to expire before the seller can rescind the contract and keep the buyer’s deposit.
The seller is not entitled to rescind the contract or keep the buyer’s deposit under the SCS until a notice to complete has been served and the notice period has expired.
The seller can rescind the contract once the notice to complete has expired but rescission is not automatically triggered by the expiry of the 10 working day notice period in the notice to complete.)

123
Q

You are acting on behalf of the seller of a residential property. Contracts were exchanged two weeks ago, and the completion date in the contract is this coming Friday.

The buyer’s solicitor has just telephoned you to explain that the buyer’s money has been delayed and the money will not now be available until the Monday following the contractual completion date.

The contract has been exchanged using the Standard Conditions of Sale (SCS)

Which one of the following correctly sets out the advice that you would now give your seller client?

A) Provided the buyer completes by 2pm on Monday and your client serves a notice to complete under the SCS, your client is entitled to three days of compensation under the SCS.

B) Provided the buyer completes by 2pm on Monday your client is entitled to one day of compensation under the SCS.

C) Provided the buyer completes by 2pm on Monday your client is entitled to three days of compensation under the SCS.

D) Time is of the essence. Your client may simply refuse to complete the sale to the buyer on Monday. The seller is entitled to keep the deposit paid by the buyer at exchange.

A

C) Provided the buyer completes by 2pm on Monday your client is entitled to three days of compensation under the SCS

(You correctly identified that the seller, who is not in default, would be able to claim compensation under the SCS for three days (Friday, Saturday and Sunday - the days of default). The compensation is payable for each day that the buyer does not complete by 2pm (the latest time for completion under the SCS), including the weekend.
Under the SCS, time is not of the essence until a notice to complete has been served. The seller would not be entitled to take the buyer’s deposit until the expiry of the notice to complete and rescission by the seller.
There is no need to serve a notice to complete in order to trigger the right to compensation, although it is common in practice for a non-defaulting party’s solicitor to serve the notice to complete to protect their client’s right to keep the deposit and rescind the contract after ten working days following the service of the notice (excluding the day the notice was given).)

124
Q

Due to an issue with their related purchase, a seller cannot complete until five calendar days following the completion date. The contract incorporates the Standard Conditions of Sale with no amendment.

The buyer had already left their rented accommodation, and has had to pay hotel and storage costs totalling £750. Assume that the costs are reasonable.

The contractual compensation for the delay would work out at £200.

What is the most the the buyer can claim from the seller?

A) The hotel and storage costs of £750.

B) Nothing, as the compensation is fault-based, and the seller is not at fault, as the delay is due to its own seller.

C) The contractual compensation of £200.

D) The hotel and storage costs of £750 less the contractual compensation of £200 = £550.

A

A) The hotel and storage costs of £750.

125
Q

Which one of the following is the correct deadline for the Land Registry application post-completion of the purchase of a registered freehold property?

A) 30 working days from the date of the OS1 priority search result.

B) 30 days from the date of completion.

C) 30 days from the date of the OS1 priority search.

D) 30 working days from the date of completion.

E) 30 days from the date contracts are exchanged.

A

A) 30 working days from the date of the OS1 priority search result

(It is 30 working days rather than 30 days. This is the priority period of the OS1 priority search and the application to the Land Registry must be made before the priority period ends.
The deadline runs from the date of the OS1 priority search result rather than completion as it is the OS1 search result which provides the priority period within which the Land Registry application must be made.
The deadline runs from the date of the OS1 priority search result made pre-completion rather than the date contracts are exchanged as it is the OS1 search result which provides the priority period within which the Land Registry application must be made)

126
Q

A company buyer has completed the purchase of a commercial freehold registered property in England with the aid of a mortgage. The seller did not have a mortgage/charge over the property. Post-completion, which one of the following is the most accurate list of the documents which the buyer’s solicitor must send to the Land Registry with their application on Form AP1?

A) The contract of sale, SDLT1, Buyer’s mortgage/charge and the Certificate of Registration of the Charge.

B) TR1, a plan of the property, SDLT5 and Buyer’s mortgage/charge.

C) TR1, SDLT5, Buyer’s mortgage/charge and the Certificate of Registration of the Charge.

D) TR1, SDLT1, Buyer’s mortgage/charge and the Certificate of Registration of the Charge.

E) The contract of sale, a plan of the property, SDLT1 and buyer’s mortgage/charge.
Correct

A

C) TR1, SDLT5, Buyer’s mortgage/charge and the Certificate of Registration of the Charge.

(The SDLT1 is the form sent to HMRC when SDLT is paid. HMRC will send back the SDLT5 acknowledgement of receipt which then needs to be sent with the Land Registry application.
The Land Registry do not need the contract of sale, they need the purchase deed (the transfer – TR1) to register the change in registered proprietor (owner) of the land.
The Land Registry do not need a plan of the property as it is already registered so they have a title plan already.
As the buyer is a company, their mortgage/charge needs to be registered at Companies House post-completion. Companies House will return a Certificate of Registration of Charge and this needs to be sent to the Land Registry as well as the buyer’s mortgage/charge so that it is registered against the title)

127
Q

A company is buying a registered freehold property in Wales with the aid of a mortgage. Which one of the following most accurately identifies the steps the buyer’s solicitor will carry out post-completion of the purchase?

A) 1) Draft the TR1,
2) submit requisitions on title,
3) carry out an OS1 search and
4) obtain finance from the buyer and its lender.

B) 1) Send the SDLT1 and pay SDLT at HMRC and
2) apply to the Land Registry using Form AP1.

C) 1) carry out an OS1 search,
2) send the SDLT1 and pay SDLT at HMRC and
3) apply to the Land Registry using Form AP1.

D) 1) Register the buyer’s mortgage at Companies House,
2) send the LTT Return and pay LTT at WRA and
3) apply to the Land Registry using Form AP1.

E) 1) Register the buyer’s mortgage at Companies House,
2) send the SDLT1 and pay SDLT at HMRC and
3) apply to the Land Registry using Form AP1.

A

D) 1) Register the buyer’s mortgage at Companies House,
2) send the LTT Return and pay LTT at WRA and
3) apply to the Land Registry using Form AP1

(As the property is in Wales LTT is payable to the WRA rather than SDLT to HMRC. The buyer’s mortgage must also be registered at Companies House (or it will be void as against liquidators, administrators and any other creditors) as the buyer is a company. The Land Registry application is necessary as the transfer and the buyer’s mortgage need to be registered on the title to the property)

128
Q

An individual buyer purchases a freehold residential registered property in England with the aid of a mortgage. The buyer did not occupy the property prior to completion. The seller did not have a mortgage over the property. Which one of the following most accurately sets out the relevant post-completion steps and their deadlines?

A) 1) Payment of SDLT at HMRC within 14 days of completion and
2) application to the Land Registry to register the transfer within 30 days of completion.

B) 1) Payment of SDLT at HMRC within 14 days of completion and
2) application to the Land Registry to register the transfer and the buyer’s mortgage within 30 working days of the OS1 priority search result.

C) 1) Registration of the buyer’s mortgage at Companies House within 21 days from completion,
2) payment of LTT at WRA within 30 days from the day after completion and
3) application to the Land Registry to register the transfer and the buyer’s mortgage within 30 days of completion.

D) 1) Registration of the buyer’s mortgage at Companies House within 21 days starting on the day after the date of completion,
2) payment of LTT at WRA within 30 days from the day after completion and
3) application to the Land Registry to register the transfer and the buyer’s mortgage within 30 working days of the OS1 priority search result.

E) 1) Registration of the buyer’s mortgage at Companies House within 21 days starting on the day after the date of completion,
2) payment of SDLT at HMRC within 14 days of completion and
3) application to the Land Registry to register the transfer and the buyer’s mortgage within 30 working days of the OS1 priority search result.

A

B) 1) Payment of SDLT at HMRC within 14 days of completion and
2) application to the Land Registry to register the transfer and the buyer’s mortgage within 30 working days of the OS1 priority search result.

(SDLT rather than LTT is relevant as the property is in England and, as there was no early occupation by the buyer, the effective date which the 14 day deadline runs from is completion.
The buyer’s mortgage does not need to be registered at Companies House because the buyer is an individual not a company.
The deadline for the Land Registry application is 30 working days from the OS1 priority search result and, as well as registering the transfer, the application to the Land Registry would include applying to register the buyer’s mortgage at the Land Registry)

129
Q

Which one of the following is the most accurate summary of the consequences of failing to comply with the relevant post-completion step within its relevant deadline following the purchase of a registered freehold property with the aid of a mortgage?

A) The consequence of failing to pay SDLT/LTT within the relevant deadline is that only the beneficial title and not the legal title passes to the buyer.

B) The consequence of failing to apply to the Land Registry to register the transfer and the buyer’s mortgage within the relevant deadline is that the transfer is void.

C) The consequence of failing to register a buyer’s mortgage at Companies House within the relevant deadline is a fine.

D) The consequences of failing to pay SDLT/LTT within the relevant deadline are penalties and interest.

E) The consequences of failing to apply to the Land Registry to register the transfer and the buyer’s mortgage within the relevant deadline are 1) that the transfer is voidable and 2) that the buyer would take the property subject to any third party interests that may have been registered subsequent to its OS1 search.

A

D) The consequences of failing to pay SDLT/LTT within the relevant deadline are penalties and interest

(The consequences of failing to register a buyer’s mortgage at Companies House within the relevant deadline are that 1) the mortgage/charge is void against liquidators, administrators and other creditors; 2) the loan is repayable on demand; and 3) it could be construed as negligence on the part of the solicitor.
The consequences of failing to apply to the Land Registry to register the transfer and the buyer’s mortgage within the relevant deadline are that 1) the buyer would take the property subject to any third party interests that may have been registered subsequent to its OS1 search; and 2) the buyer will only have beneficial title (not legal title) to the property until they are registered as the registered proprietor of the title at the Land Registry.)

130
Q

You act for a lottery winner who has decided to sell their existing house and buy a new house for £1 million. They will own no other property after completing the purchase.

At the time of completion, the SDLT rates for residential property are as follows:

- Up to £250,000: 0%

- £250,000 to 925,000: 5%

- £925,000 to £1.5 million: 10%

- Over £1.5 million: 12%

How much will you advise the buyer they will have to pay in SDLT on the new property?

A) £100,000

B) 41,250

C) £50,000

D) Nil as they no own other property.

E) £48,000

A

B) 41,250

(Correct. There is no SDLT on the first £250,000, 5% of the next slice £250,000 up to £925,000 = £33,750, and 10% of the final slice £925,000 to £1 million = £7,500. £33,750 + £7,500 = £41,250)

131
Q

You act for the buyer of a farm for a purchase price of £800,000.

At the time of completion, the SDLT rates for non-residential property are as follows:

· Up to £150,000: 0%

· £150,000 to £250,000: 2%

· Over £250,000: 5%

How much SDLT will the buyer pay?

A) £29,500

B) £40,000

C) £16,000

D) £32,500

E) £27,500

A

A) £29,500

(Correct. There is no SDLT on the first £150,000, 2% on the next £100,000 up to £250,000 = £2,000, and 5% on the remaining £550,000 = £27,500. £2,000 + £27,500 = £29,500.)

132
Q

You act for a first time buyer buying an apartment in London for £500,000. The buyer is also paying a fair market value of £5,000 for some designer furniture that the seller has agreed to leave in the apartment.

At the proposed date for completion, the SDLT rates for residential property will be as follows:

• Up to £250,000, there is no SDLT payable.

• Over £250,000 up to £925,000, SDLT of 5% is payable

• Over £925,000 up to £1.5 million, SDLT of 10% is payable

• Over £1.5 million, SDLT of 12% is payable

First time buyer rates are 0% for the first £425,000, and 5% from £425,000 to £600,000.

How much SDLT should the buyer pay?

A) £3,750

B) £4,000

C) £15,000

D) £0

E) £15,250

A

A) £3,750

(Correct. The value allocated to contents (provided it is an honest and reasonable valuation) can be excluded from the calculation. The SDLT is calculated on £500,000 and first time buyer rates apply, so it is 0% on the first £425,000 and 5% of the remaining £75,000 = £3,750)

133
Q

You act for a first-time buyer buying a house in Wales for £275,000.

At the time of completion, the relevant LTT rates for residential property are as follows:

· Up to £225,000: 0%

· £225,000 to £400,000: 6%

How much LTT will the buyer pay?

A) £16,500

B) Nil

C) £14,250

D) £3,000

E) £6,000.

A

D) £3,000

134
Q

They sell their houses to buy a larger house together.

What is the CGT position on the sale of the two houses?

A) They can claim Private Residence Relief on both houses, but at a reduced rate.

B) They can only claim Private Residence Relief on one of the houses.

C) They cannot claim Private Residence Relief on either house.

D) They can claim Private Residence Relief on both houses as their ownership pre-dated the marriage.

A

B) They can only claim Private Residence Relief on one of the houses

(Correct. For a married couple or civil partnership, only one of the houses will be eligible for PPR)

135
Q

Which of the following statements about the basis of charge to Capital Gains Tax (CGT) is correct?

A) The amount of CGT depends on the rate of income tax an individual pays.

B) CGT is payable on the sale price of the property.

C) CGT is a tax imposed on the transferee on the transfer of property.

A

A) The amount of CGT depends on the rate of income tax an individual pays

(Correct. If CGT is payable the rate depends whether the individual paying is a basic rate or higher rate taxpayer.
SDLT/LTT are the taxes payable by a purchaser (ie, the transferee) on the transfer of property to them. CGT is a tax imposed on an individual who disposes an asset (the transferor) and makes a chargeable gain.
CGT is payable on the chargeable gain made when an asset is disposed of. The chargeable gain is calculated by taking the sale price and subtracting the purchase price, any allowable expenses and the annual exemption.)

136
Q

In which one of the following situations is CGT payable?

A) A sale at a profit of an individual’s only main home which they have occupied since buying it, being 400 square metres including the garden. One room was occupied by a lodger for some of the time.

B) A sale at a profit of an individual’s only home which they have occupied since buying it, being 400 square metres including the garden.

C) A sale at a profit on an individual’s UK holiday home used by them for two months of the year and let out for the rest of the year.

D) A gift of a property to a spouse.

A

C) A sale at a profit on an individual’s UK holiday home used by them for two months of the year and let out for the rest of the year.

(Correct. This would not fall under Private Residence Relief (for detail refer back to the element) and CGT would be payable)

137
Q

Two people each own a house which they were living in when they met. They then enter into a civil partnership, and share their time between both houses.

They sell their houses to buy a larger house together.

What is the CGT position on the sale of the two houses?

A) They can only claim Private Residence Relief on one of the houses.

B) They cannot claim Private Residence Relief on either house.

C) They can claim Private Residence Relief on both houses as their ownership pre-dated the marriage.

D) They can claim Private Residence Relief on both houses, but at a reduced rate.

A

A) They can only claim Private Residence Relief on one of the houses.

(Correct. For a married couple or civil partnership, only one of the houses will be eligible for PPR)

138
Q

Which one of the following do you think a landlord would prefer to have as a tenant?

A) Coolings Landscaping Limited - a newly formed subsidiary company of Coolings Plc. Coolings Plc is a highly profitable UK garden centre chain.

B) A person who has been running a catering company for over 10 years.

C) Abacus Limited - a newly formed company with no trading history.

D) Sainsways Plc - a UK supermarket chain established 80 years ago.

A

D) Sainsways Plc - a UK supermarket chain established 80 years ago.

Correct. A chain of companies that has been trading for 80 years is more desirable than the other options shown.

139
Q

What is the attraction of commercial property to institutional investors?

A) Compared to other investments such as equities, there is little management required.

B) It offers income, the potential for capital growth and is seen as reasonably secure.

C) Commercial property is unaffected by changes in the market.

D) Commercial property is exempt from Stamp Duty Land Tax or Land Transaction Tax.

E) It offers income and capital growth which outweigh the repair costs that the landlord will need to account for.

A

B) It offers income, the potential for capital growth and is seen as reasonably secure

(Correct. Commercial property offers all of these advantages)

140
Q

Which one of the following provisions would prevent a lease being an FRI lease?

A) A tenant’s covenant to pay annual rent but no further sums.

B) A tenant’s covenant to pay towards the cost of maintaining any areas used in common with other tenants.

C) A provision requiring the rent to be paid quarterly.

D) A tenant’s covenant to keep the premises in good and substantial repair and condition.

E) Rent review provisions.

A

A) A tenant’s covenant to pay annual rent but no further sums

(Correct. An FRI lease would expect a tenant to pay the cost of insuring the property and, if appropriate, service charge)

141
Q

A warehouse owner agrees to allow a business tenant to occupy the whole of the warehouse for a year, paying a fixed sum every month. There is nothing in writing.

What is the legal status of this arrangement?

A) It is a void tenancy as it does not comply with the necessary legal formalities.

B) It is a tenancy at will.

C) It is a void tenancy as it fails for want of certainty.

D) It is a licence to occupy the premises.

E) It is a one year oral tenancy.

A

E) It is a one year oral tenancy.

Correct. A tenancy of 3 years or under may be created orally

142
Q

The owner of a department store enters into a written agreement with a key cutter to operate a stall on the ground floor, paying a weekly fixed sum. The agreement is for a minimum of six months, after which it can be ended by a month’s notice by either party. The agreement also states that the department store can require the stall to be moved to a different location as long as it is on the ground floor.

What legal arrangement is this?

A) A licence to occupy.

B) A fixed term tenancy.

C) A periodic tenancy.

D) A bare licence.

E) A tenancy at will.

A

A) A licence to occupy

(Correct. The nature of the agreement is that the key cutter may be required to move, and therefore does not have exclusive possession of any part of the premises. It is a licence)

143
Q

A solicitor is adapting a 1 year lease precedent to a term for four years.

What difference should the solicitor be aware of?

A) As the lease term is over 1 year, it will need to be signed under hand.

B) A four year lease will require more obligations on the part of the tenant.

C) As the lease term is over 3 years, it will need to be executed as a deed.

D) As the lease term is over 1 year, it will need to be executed by deed.

E) As the lease term is over 3 years, it will need to be signed under hand.

A

C) As the lease term is over 3 years, it will need to be executed as a deed

(Correct. A legal tenancy of over 3 years must be created by deed)

144
Q

Which of these statements is correct?

A) The term commencement date can only be the same day as the date of the lease.

B) The term commencement date can only be the same day or later than the date of the lease.

C) The term commencement date can only be earlier or the same day as the date of the lease.

D) The term commencement date is fixed, but the lease can be backdated, for example, to before an increase in Stamp Duty Land Tax rates.

E) The term commencement date can be earlier, the same day, or later than the date of the lease.

A

E) The term commencement date can be earlier, the same day, or later than the date of the lease.

Correct. The term commencement date can be any of these.

145
Q

A tenant has a 10 year FRI lease. The initial rent was £40,000 per annum. The 5 year upwards only rent review is coming up. The tenant’s surveyor has advised that although the rental market has stayed static, due to the landlord’s neglect of the common areas, the rent should only be £35,000 per annum on review. The hypothetical lease assumes only that the tenant, not the landlord, has complied with its covenants.

What would you expect to happen on review?

A) The rent will be reviewed to £35,000 because the hypothetical lease only applies to rent increases.

B) The rent will remain the same because the landlord’s neglect of the common areas is ignored.

C) The rent will remain the same because it is an upwards only rent review.

D) The parties are likely to negotiate the rent to somewhere between £35,000 and £40,000.

E) The rent will be reviewed to £35,000 to take account of the landlord’s neglect.

A

C) The rent will remain the same because it is an upwards only rent review

Correct. The hypothetical lease would take into account the landlord’s neglect of the common areas, but the rent will not drop because the rent review is upwards only.

146
Q

A tenant has a restaurant business in premises on a 10 year lease. The 5 year rent review is coming up. The initial rent was £10,000 per annum. The rent on the open market now would be £14,000. However, due to the tenant’s success with the restaurant, the tenant’s surveyor considers that a rival restaurant business would gladly pay a rent of £20,000 per annum for the premises. The upwards only open market rent review clause contains a standard set of assumptions and disregards. The retail prices index has increased by 5% since the beginning of the lease.

What would you expect the reviewed rent to be?

A) £14,500

B) £20,000

C) £10,500

D) £14,000

E) £10,000

A

D) £14,000

(Correct. The rent will be increased to the open market rent. The increase due to the tenant’s business (goodwill) would normally be disregarded)

147
Q

In which of the following scenarios would the Code for Leasing Business Premises apply?

A) A 10 year lease of office premises negotiated by an RICS regulated agent.

B) A six month tenancy of a warehouse negotiated by an RICS regulated agent.

C) A 10 year lease of an office premises let by a landlord who is a member of the British Landlord Association.

D) A one year lease of a flat to a couple negotiated by an RICS regulated agent.

E) A 20 year advertising hoarding licence negotiated by an RICS member.

A

A) A 10 year lease of office premises negotiated by an RICS regulated agent

(Correct. The agent would be bound by the provisions of the Code)

148
Q

Which of the following most accurately states the difference between mandatory requirements and good practice in the Code for Leasing Business Premises?

A) When the Code applies, RICS members and regulated firms must follow mandatory requirements. They must follow good practice unless there are exceptional reasons for not doing so.

B) Mandatory requirements are legally binding; good practice is advisory only.

C) When the Code applies, RICS members and regulated firms must follow mandatory requirements. They should follow good practice, but this is a matter for their discretion.

D) When the Code applies, RICS members and regulated firms must follow mandatory requirements unless there are exceptional reasons not to do so. They should follow good practice unless there are good reasons not to do so.

E) RICS members must follow mandatory requirements and good practice. Non-RICS members are only required to observe good practice.
Correct

A

A) When the Code applies, RICS members and regulated firms must follow mandatory requirements. They must follow good practice unless there are exceptional reasons for not doing so.

149
Q

A solicitor is instructed on a 10 year commercial letting by a landlord who is a member of RICS. The letting agent is unregulated. On receiving the heads of terms, the solicitor notices one provision that does not comply with the mandatory requirements of the Code for Leasing Business Premises.

Which statement best describes the solicitor’s duties?

A) The solicitor can disregard the Code as the Code only applies to the landlord.

B) The solicitor should advise the landlord of the provision that does not comply.

C) The Code would not apply as the letting agent is not regulated.

D) The solicitor must comply with the Code for Leasing Business Premises, and if the client does not agree, then the solicitor must cease acting.

E) The solicitor can assume that the Code for Leasing Business Premises applies, and may disregard the provision in the heads of terms that contravenes it.

A

B) The solicitor should advise the landlord of the provision that does not comply

(Correct. Although the landlord is an RICS member and should be aware of the Code, having spotted the provision, the solicitor should advise the landlord)

150
Q

When should pre-contract searches be carried out in a leasehold transaction?

A) They must be carried out in every leasehold transaction.

B) Searches should be unnecessary if the landlord provides sufficient detail in the replies to CPSE enquiries.

C) Searches are only necessary if the lease is registrable at the Land Registry.

D) They are required if there is a full repairing obligation.

E) They should usually be carried out, but the client may decide not to bother in some instances (eg, a short tenancy).

A

E) They should usually be carried out, but the client may decide not to bother in some instances (eg, a short tenancy).

Correct. The starting point is that searches should be carried out. If the client decides not to bother, then the solicitor should ensure that they are advised on possible consequences.

151
Q

You act for a tenant taking a 5 year lease of part from a landlord with a registered freehold title. Which is the correct pre-completion search to carry out?

A) A central land charges search against the name of the new tenant.

B) An OS3 search.

C) An OS1 search.

D) A central land charges search against the landlord’s name.

E) An OS2 search

A

B) An OS3 search

Correct. As the lease is not registrable, priority is not needed, and an OS3 search is appropriate.

152
Q

Which of the following correctly describes what a landlord’s and tenant’s solicitors do on completion of a commercial lease?

A) The landlord’s solicitor dates the counterpart lease. The tenant’s solicitor sends the completion monies and dates the original lease.

B) The landlord’s solicitor dates the original lease and the tenant’s solicitor dates the counterpart lease. The parties exchange the agreement for lease under Formula B.

C) The landlord’s solicitor dates the original lease and the tenant’s solicitor dates the counterpart lease. The tenant will usually send the completion monies to the landlord directly.

D) The landlord’s solicitor dates the original lease. The tenant’s solicitor sends the completion monies and dates the counterpart lease.

E) The landlord’s and tenant’s solicitors date the lease in duplicate and then send them to their respective clients for signature.

A

D) The landlord’s solicitor dates the original lease. The tenant’s solicitor sends the completion monies and dates the counterpart lease

Correct. The original lease is that signed by the landlord; the counterpart by the tenant. The completion monies are paid by the tenant to the landlord through their solicitors.

153
Q

A tenant takes an FRI lease of a warehouse that comprises the whole of the landlord’s title.

How is it likely that the lease will deal with insurance?

A) The landlord insures the warehouse and recovers the whole of the premium from the single tenant.

B) The landlord will insure the warehouse at its own expense, as there are no other tenants to contribute.

C) The landlord will insure the freehold and the tenant will insure the leasehold.

D) The landlord will insure the warehouse, but will contribute a small percentage of the insurance premium given that the landlord is the freeholder.

E) The tenant insures the warehouse at its own cost, and there is an obligation in the lease to this effect.

A

A) The landlord insures the warehouse and recovers the whole of the premium from the single tenant

(Correct. The landlord will normally insure, recovering the premium from the tenant)

154
Q

A tenant is taking a 10 year FRI lease of a newly constructed building. The lease excludes inherent and structural defects from the tenant’s repairing obligation.

Why does the tenant instruct its surveyor to carry out a full building survey?

A) Because it is quicker to obtain a survey than a full set of searches.

B) Because the survey will identify any problems arising from poor design or construction that will be the tenant’s responsibility in the lease.

C) Because the tenant cannot be required to put the premises into any better state of repair than it was at the date of the lease, and the survey will provide evidence of this.

D) Because under the principle of caveat emptor, the tenant cannot complain about any wants of repair that come to light after completion.

E) Because the tenant will be responsible for putting right any items that are in disrepair (save where arising from poor design or construction).

A

E) Because the tenant will be responsible for putting right any items that are in disrepair (save where arising from poor design or construction).

Correct. In an FRI lease, the tenant gives a full repairing obligation and will have to put the premises in good repair.

155
Q

A solicitor is reviewing a draft lease to work out what the tenant is directly responsible for repairing.

Where should the solicitor look for this information?

A) The tenant’s repairing and alterations covenants.

B) The tenant’s repairing covenant only.

C) The tenant’s survey.

D) The tenant’s repairing covenant and the service charge provisions.

E) The tenant’s repairing covenant and the definition of the premises.

A

E) The tenant’s repairing covenant and the definition of the premises.

Correct. The tenant’s repairing covenant is read in conjunction with the definition of the premises.

156
Q

A woman is selling a registered freehold property that she owns as tenants in common with a man. The man dies between exchange and completion. The buyer of the property will give an indemnity covenant in relation to positive covenants.

Which one of the following best describes the form of transfer deed that should be drawn up and the correct parties who must execute it?

A) A conveyance will be executed as a deed by (1) the woman and a personal representative and (2) by the buyer.

B) A conveyance will be executed as a deed by (1) the woman selling and a second trustee and (2) by the buyer.

C) A TR1 will be executed as a deed by (1) the woman selling provided she is also a personal representative and (2) by the buyer.

D) A TR1 will be executed as a deed by (1) the woman selling and a personal representative and (2) by the buyer.

E) A TR1 will be executed as a deed by (1) the woman selling and a second trustee and (2) the buyer.

A

E) A TR1 will be executed as a deed by (1) the woman selling and a second trustee and (2) the buyer.

Correct. A TR1 is the correct form of purchase deed for a registered freehold property. When there is a sole surviving tenant in common selling, a second trustee must be appointed and execute the purchase deed with the sole surviving tenant in common for the deceased tenant in common’s beneficial interest to be overreached. The buyer must execute as they are giving an indemnity covenant.

157
Q

You act for the seller of a residential property, and have exchanged with completion to take place by 3 pm today (which is a Friday). The contract incorporates the Standard Conditions of Sale. The completion monies do not arrive from the buyer until 4.45 pm. There are no bank holidays in the next few weeks.

The seller was not living in the property and does not have a related transaction, and is not put to any practical inconvenience. However, it is too late to send the money to the bank to redeem the seller’s mortgage.

What should you do?

A) Claim three days’ interest from the buyer in accordance with the contract.

B) Claim one working day’s interest from the buyer under the contract.

C) Advise your client that there is nothing that can be done as the money was received before you had a chance to serve notice to complete.

D) Claim four days’ interest from the buyer in accordance with the contract.

A

A) Claim three days’ interest from the buyer in accordance with the contract

Correct. When the contractual completion time is missed, completion is deemed to take place the next working day. This interest should compensate the seller for the additional interest that the seller will have to pay on the mortgage over the weekend.

158
Q

You act on the purchase of commercial property. The purchase is part-funded by a commercial mortgage offered by a specialist lender, who has its own solicitor acting.

Which of the following options best reflects what you and the lender’s solicitor will do after exchange but before completion?

A) The lender’s solicitor will issue a draft legal charge for approval.

B) You will provide a draft form certificate of title for approval and undertakings relating to the mortgage money to the lender’s solicitor.

C) You will request a redemption statement from the lender’s solicitor to enable completion to take place.

D) You will provide a final form certificate of title and undertakings relating to the mortgage money to the lender’s solicitor.

E) You will provide a final form certificate of title, and the lender’s solicitor will undertake to send you the mortgage money following completion.

A

D) You will provide a final form certificate of title and undertakings relating to the mortgage money to the lender’s solicitor.

Correct. The final form of certificate of title will be in the form of a draft which should have been approved by the lender/their solicitor before exchange. The lender’s solicitor will require undertakings to ensure that you can only use the mortgage money for the appropriate purpose.

159
Q

A company bought the whole of a registered freehold commercial property for £275,000.

Assume that the following rates applied at the date of completion:

0% on the first £150,000

2% on the next £100,000

5% on the next £250,000

Which one of the following best describes the post-completion steps in respect of Stamp Duty Land Tax (‘SDLT’) which the buyer’s solicitor should have carried out?

A) The buyer’s solicitor should have sent Form SDLT1 to HM Revenue and Customs within 14 days of the date of exchange together with the sum of £3,250 in respect of the SDLT payable.

B) The buyer’s solicitor should have sent Form SDLT5 to Her Majesty’s Revenue and Customs within 14 days of the date of completion together with the sum of £3,250 in respect of the SDLT payable.

C) The buyer’s solicitor should have sent Form SDLT1 to HM Revenue and Customs within 14 days of the date of completion together with the sum of £3,250 in respect of the SDLT payable.

D) The buyer’s solicitor should have sent Form SDLT1 to HM Revenue and Customs within 14 days of the date of completion together with the sum of £5,500 in respect of the SDLT payable.

E) The buyer’s solicitor should have sent Form SDLT1 to HM Revenue and Customs within 14 working days of the date of completion together with the sum of £3,250 in respect of the SDLT payable.

A

C) The buyer’s solicitor should have sent Form SDLT1 to HM Revenue and Customs within 14 days of the date of completion together with the sum of £3,250 in respect of the SDLT payable

Correct. The SDLT1 needed to be sent to HMRC within 14 days of the completion date together with the correct SDLT payable. Applying the rates that were given:
· 0% on the first £150,000 = £0
· 2% on the next £100,000 = £2,000
· 5% on the remaining amount (the portion above £250,000) i.e. the final £25,000 = £1,250
· Total SDLT = £3,250

160
Q

A man is buying the whole of a registered freehold residential property with the aid of a mortgage. The buyer’s solicitor is also acting for the buyer’s lender. The seller is a company.

Which one of the following best describes the pre-completion searches which the buyer’s solicitor should carry out?

A) An OS1 against the property and a company search against the seller.

B) An OS2 against the property, a K16 against the buyer and a company search against the seller.

C) An OS1 against the property, a K16 against the buyer and a company search against the seller.

D) An OS1 against the property and a K16 against the buyer.

E) An OS2 against the property and a company search against the seller.

A

C) An OS1 against the property, a K16 against the buyer and a company search against the seller.

Correct. An OS1 is the relevant priority search for the whole of registered land (OS2 is for sale of part but this is a sale of whole). A K16 needs to be carried out against the buyer because this is the relevant solvency search against the buyer when the buyer is an individual and the buyer’s solicitor should carry it out as they are acting for the buyer’s lender. A company search needs to be carried out against the seller as this is the relevant solvency search when the seller is a company. As this search does not give a priority period it must be carried out at the pre-completion stage even if it was also carried out earlier in the transaction.

161
Q

What effect do the mandatory provisions of Royal Institution of Chartered Surveyors (RICS) Code for Leasing Business Premises (1st edition February 2020) have regarding leasehold transactions?

A) Letting agents and other property professionals must comply with mandatory provisions, and non-compliance may be taken into account in regulatory or disciplinary proceedings.

B) Letting agents and other property professionals who are RICS members or in a RICS regulated firm must comply with mandatory provisions, and non-compliance may be taken into account in regulatory or disciplinary proceedings.

C) Following completion of a lease with an RICS regulated landlord, a tenant may apply to the court to strike out any provisions of the lease that do not comply with the mandatory provisions of the Code.

D) Following completion of a lease with an RICS regulated landlord, a tenant may apply to the court to strike out any provisions of the lease that do not comply with the mandatory provisions of the Code and are manifestly unreasonable.

E) Solicitors who act for an RICS member or RICS firm must comply with the Code or face disciplinary sanctions by the Solicitors Regulation Authority.

A

B) Letting agents and other property professionals who are RICS members or in a RICS regulated firm must comply with mandatory provisions, and non-compliance may be taken into account in regulatory or disciplinary proceedings.

Correct. RICS members are bound by the Code’s mandatory provisions. (The Code also contains statements of good practice, which RICS members should follow unless there is a good reason not to.)

162
Q

You are acting for a tenant who has agreed to take a 10 year lease of a unit in a shopping centre. The landlord is the freehold owner of the shopping centre.

Which one of the following best describes one of the pre-completion steps you will need to take before completion of the lease?

A) Carry out an OS1 search.

B) Request completion monies from the tenant.

C) Apply to register the lease at the Land Registry using form AP1.

D) Submit a final certificate of title to the bank.

E) Arrange for the tenant to sign the underlease.

A

B) Request completion monies from the tenant

Correct. You will need to be in funds for all completion monies due, which usually will be the first quarter’s rent, service charge and insurance rent so this can be sent to the landlord’s solicitor at completion.

163
Q

In which of the following situations would an agreement for lease not be necessary?

A) In a residential transaction for the sale of a newly constructed 999 year leasehold flat.

B) The tenant is taking a unit in a shopping centre that is currently under construction.

C) The parties to the lease are ready to proceed to completion as soon as the form of the lease has been agreed.

D) The tenant intends to take a high street premises for a restaurant, but first wants to apply for a premises licence which is essential for its business.

E) The landlord is in the process of buying an office block with a rental void, and the tenant has agreed to take a lease from the landlord once the purchase completes.

A

C) The parties to the lease are ready to proceed to completion as soon as the form of the lease has been agreed.

164
Q

What effect do the mandatory provisions of Royal Institution of Chartered Surveyors (RICS) Code for Leasing Business Premises (1st edition February 2020) have regarding leasehold transactions?

A) Following completion of a lease with an RICS regulated landlord, a tenant may apply to the court to strike out any provisions of the lease that do not comply with the mandatory provisions of the Code.

B) Letting agents and other property professionals must comply with mandatory provisions, and non-compliance may be taken into account in regulatory or disciplinary proceedings.

C) Following completion of a lease with an RICS regulated landlord, a tenant may apply to the court to strike out any provisions of the lease that do not comply with the mandatory provisions of the Code and are manifestly unreasonable.

D) Solicitors who act for an RICS member or RICS firm must comply with the Code or face disciplinary sanctions by the Solicitors Regulation Authority.

E) Letting agents and other property professionals who are RICS members or in a RICS regulated firm must comply with mandatory provisions, and non-compliance may be taken into account in regulatory or disciplinary proceedings.

A

E) Letting agents and other property professionals who are RICS members or in a RICS regulated firm must comply with mandatory provisions, and non-compliance may be taken into account in regulatory or disciplinary proceedings.

Correct. RICS members are bound by the Code’s mandatory provisions. (The Code also contains statements of good practice, which RICS members should follow unless there is a good reason not to.)

165
Q

Which of the following may not be a valid lease term?

A) A lease to run as long as the tenant firm is employed as the landlord’s accountants.

B) A 10 year lease to start one year after signing the lease.

C) A lease to run for 10 years, but may be brought to an end earlier if the tenant firm is no longer employed as the landlord’s accountants.

D) A lease in which the tenant can occupy on a month by month basis, paying the rent monthly.

E) A 10 year lease with the option to renew the lease at the end of the 10 years.

A

A) A lease to run as long as the tenant firm is employed as the landlord’s accountants

Correct. This is not a valid lease term. The lease term must be certain and may not continue indefinitely.

166
Q

In negotiations to take a commercial lease in England, the prospective tenant asks for a 5 year lease term as the tenant knows that it will not need the premises for longer. The landlord offers a 10 year lease with a 5 year mutual break clause, saying this is just as good.

As the prospective tenant’s solicitor, what advice can you give?

A) Although the tenant can get out of the lease at 5 years, exercising a break is not straightforward. However, the Stamp Duty Land Tax calculation will not be affected.

B) You cannot advise your client on this as it is regulated financial advice.

C) A 10 year lease with 5 year break is better, as it as it gives the tenant a choice whether to leave at 5 years or 10 years. The Stamp Duty Land Tax calculation will not be affected.

D) A 10 year lease with 5 year break is better, as it gives the tenant a choice whether to leave at 5 years or 10 years. However, there may be more Stamp Duty Land Tax to pay.

E) Although the tenant can get out of the lease at 5 years, exercising a break is not straightforward, and there may be more Stamp Duty Land Tax payable on the longer term.

A

E) Although the tenant can get out of the lease at 5 years, exercising a break is not straightforward, and there may be more Stamp Duty Land Tax payable on the longer term.

Correct. Break clauses are generally strictly drafted, and if the tenant does not comply exactly with their requirements, the break notice will be invalid. As the length of the lease is relevant to SDLT calculations, there may be a greater amount of SLDT to pay.

167
Q

A tenant’s solicitor is calculating the SDLT due after completion. The lease has a Net Present Value of £105,000 and no premium. The landlord’s freehold is worth £200,000. The applicable rates are 0% up to £150,000, and 1% from £150,000 up to £5 million.

How much SDLT is payable?

£1,050

£5,000

£1,500

£0

£4,500

A

£0

Correct. There is no SDLT on this Net Present Value.

168
Q

A tenant’s solicitor is calculating the LTT payable on a lease with a NPV of £425,000 and no premium. The prevailing rates are 0% up to £225,000 and 1% from £225,000 to £2 million.

What LTT is payable?

£2,250

£4,250

£2,000

£0

It is not possible to say, as it will depend whether the rent is payable annually, quarterly or monthly.

A

£2,000

Correct. The LTT is calculated as 1% of the £200,000 by which the NPV exceeds £225,000.

169
Q

A tenant’s solicitor is dealing with the post-completion tasks on a 5 year lease.

Which of the following summarises the position with registration of the new lease?

A) The lease must be registered as the grant of lease is an event triggering first registration.

B) The lease must be registered and will be given its own title number. It will also be noted against the landlord’s title.

C) The lease is not registrable and cannot be noted against the landlord’s title.

D) The lease must be registered if the landlord’s title is registered.

E) The lease is not registrable but can be noted against the landlord’s title.

A

E) The lease is not registrable but can be noted against the landlord’s title.

Correct. The lease is more than 3 years but not more than 7 years. This is the correct answer.

170
Q

A lease contains a covenant as follows: “not to alter the interior of the premises without the consent of the landlord”.

The tenant wants to insert some internal, non-structural partitioning.

How would you advise the landlord on this application?

A) The landlord can unreasonably withhold consent because if the tenant wants to make an improvement it can use the statutory procedure.

B) The landlord need not consider the application as it is an absolute covenant.

C) It is likely that the landlord cannot unreasonably withhold consent if the alterations are considered an improvement from the tenant’s point of view.

D) It is likely that the landlord cannot unreasonably withhold consent if the alterations are considered an improvement from the landlord’s point of view.

E) The landlord can unreasonably withhold consent because it is not a fully qualified covenant.

A

C) It is likely that the landlord cannot unreasonably withhold consent if the alterations are considered an improvement from the tenant’s point of view

Correct. As the alteration improves the premises from the tenant’s perspective it would class as a tenant’s improvement.

171
Q

Which of the following would be unlikely to appear in a licence for alterations?

A) A copy of the lease.

B) An obligation on the tenant to carry out work with good quality materials and a high standard of workmanship.

C) A time limit for starting and carrying out the works.

D) An obligation to obtain planning consent.

E) Plans and/or specifications of the proposed works.

A

A) A copy of the lease

Correct. It is not usually necessary to include a copy of the lease.

172
Q

A tenant occupies premises under a lease with a permitted use defined as “use as a hairdresser’s salon”. There is a qualified covenant against changes of use. The tenant wants to stop hairdressing and sell beauty products instead.

What does the tenant need to do?

A) The tenant need not ask for consent, as the permitted use definition breaches the Code for Leasing Business Premises.

B) The tenant must ask for consent, and the landlord must act reasonably if withholding consent as the new use is not dissimilar.

C) The tenant need not ask for consent, as the permitted use and proposed use are both in planning use class E.

D) The tenant must ask for consent, but the landlord may unreasonably withhold it.

E) The tenant must ask for consent, and the landlord must act reasonably if withholding consent, as this is a tenant’s improvement.

A

D) The tenant must ask for consent, but the landlord may unreasonably withhold it

Correct. As it is not a fully qualified covenant, the landlord has no obligation to give consent.

173
Q

A tenant occupies under a 3 year legal lease. The lease contains the following covenant:

‘The Tenant shall not make any internal, non-structural alteration to the Premises without the consent of the Landlord, such consent not to be unreasonably withheld.’

The tenant would like to construct an internal partition wall and also carry out some general internal fitting out works to its premises. It has written to the landlord asking for their consent to these works explaining the nature of the works and enclosing detailed plans.

Which of the following statements best explains your advice to the landlord in respect of giving a decision to the tenant?

A) The landlord cannot unreasonably withhold its consent to the works because the covenant is fully qualified.

B) The landlord has total discretion whether to consent to the works because there is an absolute covenant against alterations.

C) The landlord unreasonably withhold consent provided the proposed works do not constitute tenant’s improvements.

D) The landlord has total discretion whether to consent to the works because the covenant against alterations is qualified.

E) The landlord cannot unreasonably withhold its consent to the works because s. 19(2) of the Landlord and Tenant Act 1927 applies to this covenant.

A

A) The landlord cannot unreasonably withhold its consent to the works because the covenant is fully qualified

This is correct. The covenant against the internal alterations is fully qualified. This means the landlord cannot unreasonably withhold its consent to the works. S. 19(2) of the Landlord and Tenant Act 1927 does not apply to alterations covenants that are already fully qualified.
Leases will normally allow the tenant to make some alterations to the premises. Unless the lease stipulates otherwise, the tenant is free to carry out any alterations to the premises, subject to the legal doctrine of ‘waste’ which prevents alterations which would devalue the premises. However, the landlord will usually wish to control what alterations the tenant can undertake, especially in shorter leases. You need to be able to distinguish between absolute, qualified and fully qualified covenants in a lease and understand the effect of statute on certain covenants. S.19 (2) Landlord and Tenant Act 1927 applies to certain alterations covenants. It is important you understand when it will / will not apply

174
Q

**A freeholder (the ‘Landlord’) granted a commercial lease to a tenant. The tenant currently uses the property as a hairdresser but wants to change the use to a clothes shop. The tenant does not plan to undertake any significant alterations to the property. The lease provides the following:

‘The Tenant covenants not to use the Property other than as a hairdresser or a hair salon without the consent of the Landlord.’**

Which one of the following best describes the Landlord’s position?

A) The Landlord is required to act reasonably in deciding whether or not to grant consent to the change of use.

B) The Landlord is entitled to charge a lump sum as a condition of granting consent to the change of use.

C) The Landlord is entitled to increase the rent as a condition of granting consent to reflect the new use.

D) The Landlord cannot withhold consent as the tenant’s proposed use is within the same planning use class as the existing use.

E) The Landlord will be able to withhold consent to the request to change the use of the property.

A

E) The Landlord will be able to withhold consent to the request to change the use of the property.

This is correct. S.19(3) of the Landlord and Tenant Act 1927 does not upgrade the qualified covenant into a fully qualified covenant so the Landlord does not need to act reasonably. This section does prohibit a landlord for charging a lump sum or an increase in the rent unless there are structural alterations taking place. The section also allows a landlord to require the payment of its legal fees as a condition of consent so there is no need for this obligation to be in the lease itself.

175
Q

A tenant occupies the first floor of a building under a lease. It wishes to make several alterations to the premises which include erecting several non-structural walls to split the first floor into a series of offices. The lease contains a provision prohibiting non-structural alterations without the landlord’s prior consent.

When considering the tenant’s application for consent, which statement best describes the landlord’s position?

A) The alterations provision in the lease is a qualified covenant and as the tenant’s proposed works constitute improvements, the landlord cannot unreasonably withhold its consent.

B) The alterations provision in the lease is a qualified covenant and, as the tenant’s proposed works are likely to constitute improvements, the landlord cannot unreasonably withhold its consent.

C) The alterations provision in the lease is a qualified covenant, but the landlord cannot unreasonably withhold consent if the tenant can show that the proposed works will enhance the value of the landlord’s reversion.

D) The alterations provision in the lease is a qualified covenant and therefore the landlord is entitled to unreasonably withhold its consent.

E) The alterations provision in the lease is a fully qualified covenant and, as the tenant’s proposed works constitute improvements, the landlord cannot unreasonably withhold its consent.

A

B) The alterations provision in the lease is a qualified covenant and, as the tenant’s proposed works are likely to constitute improvements, the landlord cannot unreasonably withhold its consent.

Correct. The requirement for landlord’s consent means that the alterations covenant is a qualified covenant. Section 19(2) of the 1927 Act upgrades this to a fully qualified covenant where the works constitute tenant’s improvements meaning the landlord cannot unreasonably withhold consent. As the tenant is presumably proposing the alterations to make the premises more suitable for its use, they would be likely to qualify as tenant’s improvements.

176
Q

You act for a tenant with lease of the ground floor in an office block. The premises have recently been damaged by flooding, and the tenant asks for your advice.

The repair clause in the lease states: ‘the Tenant shall keep the Premises in repair save for where any damage has been caused by an Insured Risk’.

The definition of Insured Risks includes the following wording: “fire, explosion, lightning, earthquake, tempest, storm, flood, bursting and overflowing of water tanks, apparatus or pipes, damage to underground water, oil or gas pipes or electricity wires or cables …”

Which of the following best describes the advice you would give to the tenant as to liability for repairing damage to the premises caused by flooding?

A) You would advise the tenant that the landlord will likely be liable for repairing damage to the premises caused by flooding.

B) You would explain to the tenant that you need to wait for the results of the flood risk search before you will be able to advise.

C) You advise the tenant to claim on their buildings insurance policy.

D) You advise that the insurance company is responsible for repairing damage to the premises caused by flooding.

E) You advise that the landlord, tenant and insurance company would be jointly and severally liable for repairing damage caused to the premises by flooding.

A

A) You would advise the tenant that the landlord will likely be liable for repairing damage to the premises caused by flooding

Correct. Flooding is an insured risk, and these are excluded from the tenant’s repairing obligation. It is likely (but you should check) that the landlord’s obligations are to apply the money received from the insurers on repairing the damage.

177
Q

Which of the following best reflects the structure of an FRI lease for a unit in a shopping centre?

A) The tenant is responsible for insuring the property, recovering the cost of doing so from the tenants.

B) The tenant is responsible for repairing the interior of the unit. The landlord is responsible for insuring the shopping centre. The landlord need not covenant to provide the services, provided a managing agent is appointed to carry them out.

C) The tenant will be responsible for the repair of their unit, but will only be obliged to keep it in the state of repair in which it finds it (ie, at the beginning of the lease). The landlord is responsible for insuring the property, recovering the cost of doing so from the tenants.

D) The landlord is responsible for repairing the interior of the unit, but will recover the whole cost of doing so from the tenant. The landlord is responsible for insuring the shopping centre, but will recover a proportionate part of the cost of doing so from the tenant.

E) The tenant is responsible for repairing the interior of the unit. The landlord is responsible for insuring the shopping centre and providing the services, recovering a proportionate part of the cost of doing so from the tenant.

A

E) The tenant is responsible for repairing the interior of the unit. The landlord is responsible for insuring the shopping centre and providing the services, recovering a proportionate part of the cost of doing so from the tenant.

Correct. Note that the landlord does undertake obligations in an FRI lease, but the cost falls upon the tenants.

178
Q

Which of the following best summarises the liabilities of the outgoing tenant and incoming tenant for the tenant obligations in the lease after assignment for an old lease and a new lease?

A) For an old lease, the outgoing tenant remains liable. The incoming tenant is not liable unless it gives a direct covenant to the landlord.
For a new lease, the outgoing tenant’s liability will depend whether the incoming tenant gives an authorised guarantee agreement.

B) For an old lease, the outgoing tenant remains liable. The incoming tenant automatically becomes liable.
For a new lease, the outgoing tenant remains liable. The incoming tenant only becomes liable if it gives an authorised guarantee agreement.

C) For an old lease, the outgoing tenant remains liable. The incoming tenant automatically becomes liable.
For a new lease, the outgoing tenant is released from liability unless it enters into an authorised guarantee agreement. The incoming tenant automatically becomes liable.

D) For an old lease, the outgoing tenant is released from liability unless it enters into an authorised guarantee agreement. The incoming tenant automatically becomes liable.
For a new lease, the outgoing tenant remains liable, but the incoming tenant only becomes liable if it gives a direct covenant to the landlord.

E) For an old lease, the outgoing tenant remains liable, but the incoming tenant only becomes liable if it gives a direct covenant to the landlord.
For a new lease, the outgoing tenant is released from liability unless it enters into an authorised guarantee agreement. The incoming tenant automatically becomes liable.

A

E) For an old lease, the outgoing tenant remains liable, but the incoming tenant only becomes liable if it gives a direct covenant to the landlord.
For a new lease, the outgoing tenant is released from liability unless it enters into an authorised guarantee agreement. The incoming tenant automatically becomes liable.

Correct. This states the liabilities of each party after assignment correctly.

179
Q

In which of the following scenarios would assignment not be an appropriate option?

A tenant with a takeaway is selling its takeaway business, including kitchen equipment and goodwill.

A tenant’s business is failing, and it still has five years left to run of its lease.

The tenant does not need the premises at the moment, but may do in future.

A company is transferring all its property assets to a new holding company.

A person is selling their long leasehold flat.

A

The tenant does not need the premises at the moment, but may do in future

Correct. Assignment would not be appropriate here, as the tenant would be giving up control of the premises.

180
Q

The landlord of a shopping centre has granted one of the retail unit tenants a lease with a qualified covenant against assignment. Which of the following describes best where the landlord can withhold consent?

A) The landlord must give a reason, but it does not need to be reasonable, as it is not a fully qualified covenant.

B) The proposed assignee is a group company of the tenant.

C) The proposed assignee sells similar products to the shop next to the tenant’s.

D) For any reason or no reason at all, as it is not a fully qualified covenant.

E) The proposed assignee is a shop belonging to a religious organisation which the landlord suspects to be a cult.

A

C) The proposed assignee sells similar products to the shop next to the tenant’s.

Correct. In a shopping centre, the landlord will usually be able to impose a policy to mix the retail uses.

181
Q

A landlord will incur legal costs in considering an application for consent to assign.

Which of the following is most likely?

A) The landlord’s solicitor will raise an invoice at the end of the process, and rely on the obligation in the lease that the tenant will pay the landlord’s costs.

B) The landlord will require an undertaking for costs from the tenant.

C) The landlord’s solicitor will require money from the tenant’s solicitor to be paid upfront.

D) The landlord’s solicitor will require an undertaking for costs from the tenant’s solicitor.

E) The landlord’s solicitor will require an undertaking for costs from the prospective assignee’s solicitors.

A

D) The landlord’s solicitor will require an undertaking for costs from the tenant’s solicitor.

182
Q

You act for the buyer of a flat in a block of 65 flats, held by the current owner on a 150 year lease with 142 years left to run. The lease is registered at the Land Registry.

Which of the following would you not likely need to do?

A) Carry out an OS1 search.

B) Approve the draft contract prepared by the seller.

C) Apply to the landlord for consent to the assignment.

D) Carry out pre-contract searches.

E) Report to your client on service charge information given in replies to enquiries.

A

C) Apply to the landlord for consent to the assignment

Correct. It is not usual for long residential leases to require landlord’s consent to assignment.

183
Q

Which of the following statements about pre-completion searches is correct?

An OS1 search should be carried out for the assignee of a 5 year lease.

An OS1 search should be carried out for the assignee of a 10 year lease.

If the landlord’s title is unregistered, a Central Land Charges search should be carried out on assignment.

An OS2 search should be carried out for the assignee of a 10 year lease of part.

An OS2 search should be carried out for an assignee of a 5 year lease of part.

A

An OS1 search should be carried out for the assignee of a 10 year lease

Correct. A 10 year lease would be registered, and so an OS1 search should be carried out against the assignor’s leasehold title.

184
Q

Which of the following statement correctly describes the liabilities of the tenant and undertenant for the tenant’s lease obligations to the landlord following an underletting?

A) The tenant will remain liable and the undertenant will be liable automatically if it is a new lease.

B) The tenant will remain liable if it gives an authorised guarantee agreement and the undertenant will be liable if it gives a direct covenant.

C) The tenant will remain liable and the undertenant will be liable if the undertenant gives a direct covenant to the landlord.

D) The tenant remains liable to the landlord, but the undertenant cannot be made liable to the landlord because of privity of estate.

E) The tenant will only remain liable if it is an old lease, and the undertenant will be liable if it gives a direct covenant.

A

C) The tenant will remain liable and the undertenant will be liable if the undertenant gives a direct covenant to the landlord

Correct. The tenant remains in a landlord/tenant relationship with the landlord. The undertenant will only be liable to the landlord if it gives a direct covenant (but it will be liable to the tenant as its immediate landlord).

185
Q

A tenant of the whole of an office block under an FRI lease intends to underlet one floor to an advertising agency. Which of the following is likely to be a permissible term for an underlease?

The remainder of the term of the tenant’s lease less three days.

The remainder of the tenant’s term.

For as long as the tenant contracts its advertising work to the undertenant.

The remainder of the term of the tenant’s lease plus three days.

On a monthly periodic tenancy.

A

The remainder of the term of the tenant’s lease less three days.

Correct. This is a certain term, and is less than the tenant’s term.

186
Q

A tenant has a 10 year lease with 7 years left to run. The lease is for four floors of a city office tower. There are no break clauses. The lease contains a qualified covenant against underletting of part or a permitted part, being one or more of the four floors.

In which of the following scenarios would underletting not be an appropriate option?

The tenant’s business has moved to another premises, except for a small team, and it is only using one of the four floors. It has found someone willing to occupy two of the floors, but has not yet found anyone for the other.

The tenant wishes to be released from its repairing obligation.

The tenant’s business has downsized, and it is only using three of the four floors. It has found someone willing to occupy the other floor.

The tenant does not need the premises but cannot find an assignee.

The tenant does not need the premises for the next two years and has found someone willing to occupy for that time.

A

The tenant wishes to be released from its repairing obligation.

Correct. Underletting does not release the tenant from its repairing obligation (although it can in turn require the undertenant to comply with it).

187
Q

On an underletting of an FRI lease, which of the following statements is most accurate regarding the necessary documentation?

The tenant’s solicitor drafts the licence to underlet and the undertenant’s solicitor drafts the underlease.

The landlord’s solicitor drafts the licence to underlet and the undertenant’s solicitor drafts the underlease.

The landlord’s solicitor drafts the licence to underlet and the tenant’s solicitor drafts the underlease.

The landlord’s solicitor drafts the licence to underlet and the tenant’s solicitor drafts the underlease and transfer TR1.

The landlord’s solicitor drafts the licence to underlet and the underlease for the tenant’s approval.

A

The landlord’s solicitor drafts the licence to underlet and the tenant’s solicitor drafts the underlease.

Correct. The landlord’s solicitor will draft the licence to underlet, and it is for the tenant’s solicitor to draft the underlease.

188
Q

The tenant of a restaurant business wishes to underlet the premises to a cousin. In which of the following circumstances would an agreement for underlease not be necessary?

The tenant and undertenant want to commit to the underlease, but in the case of the undertenant, only if it can get an alcohol licence.

The parties are ready to complete the underlease but the consent application has been delayed.

The tenant and undertenant want to commit to the underlease, but only if the tenant complies with a recent hygiene improvement notice.

The undertenant only wants to take the underlease as part of a sale of the tenant’s business.

The parties are ready to complete the licence to underlet and underlease.

A

The parties are ready to complete the licence to underlet and underlease.

Correct. If the parties are ready to complete, there is no need for the agreement for underlease.

189
Q

An undertenant is taking the remaining 8 years of a tenant’s term under an FRI lease of a warehouse. The underlease is for half of the warehouse floor space.

Which is the correct pre-completion search for the undertenant’s solicitor to carry out?

An OS2 search against the tenant’s leasehold title.

An OS2 search against the landlord’s freehold title.

An OS1 search against the landlord’s freehold title.

An OS1 search against the tenant’s leasehold title.

OS3 searches against both the landlord’s freehold title and the tenant’s leasehold title.

A

An OS2 search against the tenant’s leasehold title

Correct. As the underlease is of part of the premises, an OS2 search of part should be carried out against the tenant’s leasehold title.

190
Q

Which of the following is not a function of a licence to assign an old lease?

To formally grant the landlord’s consent.

To obtain a covenant from the tenant to pay the landlord’s costs.

To require the assignee to give notice once the assignment has taken place.

To set out the credit checks and references that will be required of a prospective tenant.

To obtain a direct covenant from the new tenant.

A

To set out the credit checks and references that will be required of a prospective tenant.

Correct. These will be carried out as part of the consent application, but as they will be done before the licence to assign is completed, they would not usually be referred to in the licence itself.

191
Q

Which of the following would you not be likely to find in a licence to assign or underlet?

A direct covenant by proposed assignee or undertenant to the landlord to comply with the tenant obligations under the lease.

A covenant to pay the landlord’s costs.

A requirement that the proposed assignment or underletting takes place within a certain time, say 3 months.

For a licence to underlet, a draft of the underlease. For a licence to assign, an authorised guarantee agreement.

A detailed specification of work to be carried out.

A

A detailed specification of work to be carried out.

Correct. You would expect to find this in a licence for alterations.

192
Q

Residential conveyancers do not often deal with licences to assign. Why not?

Because the cost of a licence to assign would be disproportionate in a residential context.

Because consent to assignment of a residential lease is conferred automatically by statute.

Because a licence to assign can only apply where the proposed assignee is not paying the tenant a premium.

Because residential conveyancers do not deal with assignments of lease.

Because most long leases do not require landlord’s consent to an assignment.

A

Because most long leases do not require landlord’s consent to an assignment.

193
Q

You act for an individual landlord who has received an application to underlet one of their units to a sole proprietor. The sole proprietor has a successful business, plenty of capital and excellent tenant references and credit score. However, the landlord knows the sole proprietor from a club that they both belong to, and they have had a falling out. For that reason alone, the landlord does not want to grant the consent.

How do you advise the landlord?

A) The landlord is required to act reasonably if withholding consent, and personal feelings are not a reasonable ground for withholding consent. The landlord may, however, stall for time and hope that the proposed underletting falls through.

B) The landlord cannot unreasonably withhold consent, but given that the underlessee might end up being the direct tenant of the landlord, it is reasonable to take their personal relationship into account as it might make their business relationship unworkable.

C) The landlord is required to act reasonably if withholding consent, and personal feelings are not a reasonable ground for withholding consent. The landlord will likely have to give consent within a reasonable time.

D) The landlord can unreasonably withhold consent as it is only a qualified covenant.

E) The landlord cannot unreasonably withhold consent, but it is reasonable to withhold consent to a sole proprietor as their covenant strength is less than a company’s.

A

C) The landlord is required to act reasonably if withholding consent, and personal feelings are not a reasonable ground for withholding consent. The landlord will likely have to give consent within a reasonable time

This is a qualified covenant against underletting (the landlord’s consent is required), which is automatically upgraded to a fully qualified covenant (s19(1)(a) Landlord and Tenant Act 1927). Furthermore, the landlord must make a decision within a reasonable time (s1, Landlord and Tenant Act 1988).
As the prospective underlessee has good covenant strength, and the landlord’s only objection is their personal relationship, it is likely that the landlord will have to give consent within a reasonable time.

194
Q

A 20 year lease is dated 1 July 2010. The tenant when the lease was granted was a clothes shop. The clothes shop lawfully assigned the lease to a bookstore and entered into an authorised guarantee agreement (AGA). The bookstore, two years later, lawfully assigned the lease to a dry cleaners and entered into an AGA. The dry cleaners still occupy the premises under the lease. The dry cleaners are struggling to pay their rent and the landlord wants to know who it can pursue for the rent arrears.

Which one of the following is the most accurate list of those the landlord can pursue for the rent arrears?

A) The clothes shop only.

B) The clothes shop, the bookstore and the dry cleaners.

C) The dry cleaners only.

D) The bookstore and the dry cleaners.

E) The clothes shop and the dry cleaners.

A

D) The bookstore and the dry cleaners

The dry cleaners are the current tenant and are liable to pay the rent. The bookstore gave an AGA to the landlord in respect of the dry cleaners when they assigned to the dry cleaners. The landlord can therefore claim from the bookstore under the AGA.
The original tenant (the clothes shop) will not have any original tenant liability as this is a new lease. The clothes shop’s AGA only covered the bookstore whilst they were the tenant.

195
Q

A tenant, A Ltd, has applied for consent to assign a five year lease to the assignee, B Ltd. The lease is dated after 1 January 1996.

You act for the landlord, who has asked you to advise who would be liable for the obligations of the lease after the assignment. How would you advise?

A) Both the outgoing tenant and the incoming tenant as it is a new lease.

B) The outgoing tenant if they entered into an authorised guarantee agreement, and the incoming tenant automatically on taking the assignment.

C) The incoming tenant provided that they give a direct covenant to the landlord to observe the obligations of the lease.

D) The incoming tenant if the landlord requires an authorised guarantee agreement; otherwise the outgoing tenant retains liability.

E) The outgoing tenant unless they enter into an authorised guarantee agreement, and the incoming tenant automatically on taking the assignment.

A

B) The outgoing tenant if they entered into an authorised guarantee agreement, and the incoming tenant automatically on taking the assignment

Correct. This is a new lease, and therefore liability shifts automatically from the assignor to the assignee on assignment. The outgoing tenant will only be liable if they have given an authorised guarantee agreement, but the incoming tenant will become automatically liable on assignment.

196
Q

A tenant has a registered lease dated before 1 January 1996. The tenant is assigning the lease to an assignee. The landlord has consented to the assignment and has asked you to draft the relevant documentation.

Which one of the following is the correct list of the documents that will be need to be prepared for execution?

A) Licence to assign, authorised guarantee agreement and the lease.

B) Licence to assign and deed of assignment.

C) Licence to assign and the lease.

D) Licence to assign, authorised guarantee agreement and TR1

E) Licence to assign and TR1

A

E) Licence to assign and TR1.

A licence to assign is needed to document the landlord’s consent.
As the lease is registered, the assignment will need to be documented by way of Land Registry’s standard transfer deed TR1.
An assignment is the sale of an existing lease, not the creation of a new lease. There will be no new lease granted to the assignee.

197
Q

You act for an assignee taking the remaining 3 years of a 10 year term. The assignment is about to complete. The immediate landlord of the current tenant is the freeholder.

Which pre-completion searches should you undertake?

An OS2 search of the relevant premises against the landlord’s freehold title.

An OS1 search of the landlord’s freehold title (or if the landlord’s title is unregistered a central land charges search against the landlord’s name for the landlord’s period of ownership), but as there is only 3 years left on the lease, no search against the leasehold.

An OS1 search of the tenant’s leasehold title.

An OS3 search as the assignment is for 3 years and therefore not registrable.

An OS1 search of the landlord’s freehold title (or if the landlord’s title is unregistered a central land charges search against the landlord’s name for the landlord’s period of ownership) and an OS1 search against the tenant’s leasehold title

A

An OS1 search of the tenant’s leasehold title.

Correct. As the original lease was over 7 years, it would have been registered. It is the tenant’s title that the assignee is interested in.

198
Q

Which of the following is not a benefit of security of tenure for the landlord?

The landlord may find it easier to attract good tenants.

The landlord may be able to show that it has complied with the Code for Leasing Business Premises.

Security of tenure provides the landlord with greater flexibility over its own property.

Security of tenure might encourage the tenant to treat the premises as their own and therefore look after them.

The reviewed rent may be higher for a secured tenancy.

A

Security of tenure provides the landlord with greater flexibility over its own property.

Correct. This is not a benefit - the landlord is more restricted in its use of the property.

199
Q

Which of the following arrangements could qualify for security of tenure under the Landlord and Tenant Act 1954?

A licence for a coffee seller to take a stall in a department store.

A six month tenancy of a warehouse unit with no previous occupation.

A monthly tenancy for a shop with a flat above it.

A one year assured shorthold tenancy for a family to occupy house.

A tenancy at will of an office building pending negotiations of a final lease.

A

A monthly tenancy for a shop with a flat above it.

Correct. A periodic tenancy can qualify for security of tenancy. The flat does not prevent the premises from qualifying as it is a subsidiary residential use.

200
Q

A farmer lets a grazing field and cowshed to a neighbouring farmer for a term of twelve months. The farmer does not go through the contracting out procedure of the Landlord and Tenant Act 1954.

Which of the following statements regarding security of tenure is most accurate?

The tenancy would not have security of tenure under the Landlord and Tenant Act 1954 because it is only a 12 month term.

The tenancy would have security of tenure under the Landlord and Tenant Act 1954 provided the neighbouring farmer uses the land for its farming business.

The tenancy does not have security of tenure under the Landlord and Tenant Act 1954 because it is an agricultural tenancy.

The tenancy would not have security of tenure under the Landlord and Tenant Act 1954 because it is a service tenancy.

The tenancy would have security of tenure under the Landlord and Tenant Act 1954 because the contracting out procedure wasn’t followed.

A

The tenancy does not have security of tenure under the Landlord and Tenant Act 1954 because it is an agricultural tenancy.

Correct. As an agricultural tenancy it would not qualify for security of tenure under the Landlord and Tenant Act 1954. It may qualify for a different type of security of tenure under the agricultural tenancy regime, but that is beyond the scope of this course.

201
Q

A tenant has agreed to take a lease contracted out of the security of tenure provisions of the Landlord and Tenant Act 1954.

The landlord served notice of contracting out 7 days ago and completion is taking place today. What is needed to ensure that the lease is validly contracted out?

An ordinary declaration and reference to both the notice and statutory declaration in the lease itself.

As insufficient time has elapsed, a court order approving the contracting out.

A statutory declaration and reference to both the notice and statutory declaration in the lease itself.

A statutory declaration and reference to the statutory declaration in the lease itself.

An ordinary declaration and a statutory declaration and reference to both the notice and statutory declaration in the lease itself.

A

A statutory declaration and reference to both the notice and statutory declaration in the lease itself.

Correct. As 14 days have not elapsed since the landlord’s notice, a statutory declaration is required. It is also necessary to refer to both notice and statutory declaration in the lease itself.

202
Q

A 10 year FRI lease is about to reach the expiry of its contractual term. It is protected by the security of tenure provisions of the Landlord and Tenant Act 1954.

How would you advise the tenant?

The tenant can leave on the last day of the contractual term, or, if it has served a section 26 notice or received a friendly section 25 notice, can remain in occupation.

The tenant can leave the premises on the last day of the contractual term, or it can remain in occupation beyond the last day.

The tenant can leave the premises on the last day of the contractual term, or, provided it gives the landlord warning, can remain in occupation beyond the last day.

The tenant can leave the premises on the last day of the contractual term provided it has given the landlord notice to that effect, or if not, can remain in occupation beyond the last day.

The tenant can leave the premises on the last day of the contractual term, or any day after the last day and need not give notice.

A

The tenant can leave the premises on the last day of the contractual term, or it can remain in occupation beyond the last day.

Correct. The tenant is free to treat the contractual expiry as the end of its occupation, or to remain in occupation and hold over.

203
Q

The tenant of a 10 year FRI lease (protected by the security of tenure provisions of the Landlord and Tenant Act 1954) is in substantial rent arrears.

How would you advise the landlord?

The landlord may only forfeit the lease after the expiry of the contractual term.

The landlord may forfeit the lease.

The landlord may not forfeit the lease as it is protected.

Forfeiture is available to the landlord provided it serves a hostile section 25 notice.

The landlord may only forfeit the lease with the leave of the court as it is protected.

A

The landlord may forfeit the lease.

Correct. The landlord may forfeit the lease even if it is protected.

204
Q

A surveyor’s firm takes a quarterly periodic tenancy of an office. The tenancy is protected.

The surveyor’s firm wishes to vacate the premises. How would you advise them?

They can simply vacate the premises at any time.

They will need to serve a section 27 notice.

They will need to serve a section 26 notice on the landlord.

They will need to negotiate a surrender of the lease with the landlord.

They may serve notice to quit on the landlord.

A

They may serve notice to quit on the landlord.

Correct. This is the correct procedure (more on this in the materials on termination).

205
Q

When the major supermarket chains take a protected lease of premises (eg, for local supermarket branches), they may make it a condition of the lease that the landlord does not assign the reversion of the lease to a rival supermarket chain.

Why is this?

Because if the rival supermarket becomes the landlord, then after five years of holding the reversion, they may oppose renewal on the basis of occupying the premises for their own use.

Because supermarket chains like to make things difficult for other supermarkets.

Because if the rival supermarket becomes the landlord, then they may oppose renewal at the next opportunity (no matter when) on the basis of occupying the premises for their own use.

Because a supermarket business cannot be landlord to another supermarket business.

Because the rival supermarket will make the landlord/tenant relationship so strained that the tenant supermarket will give up occupation at the next available opportunity.

A

Because if the rival supermarket becomes the landlord, then after five years of holding the reversion, they may oppose renewal on the basis of occupying the premises for their own use.

Correct. This has happened in practice, and therefore this is why supermarkets may want to include a clause of this type

206
Q

In which of the following circumstances would a tenant serve a section 26 notice?

The tenant wants to take advantage of a falling rental market.

The landlord has served a friendly section 25 notice.

The landlord has served a hostile section 25 notice.

The tenant is intending to leave the premises shortly.

The tenant wants to take advantage of a rising rental market.

A

The tenant wants to take advantage of a falling rental market.

Correct. This is a good reason, as the tenant will otherwise be stuck on the rent last fixed by the previous lease.

207
Q

Which of the following is not a valid use of a section 27 notice?

The tenant serves a s27 notice to leave on the last day of the contractual term.

The tenant is holding over and wishes to leave the premises in three months’ time.

The tenant serves a s27 notice to leave the week before the end of the contractual term.

The tenant intended to leave the premises on the last day of the term, but miscalculated the dates, and now gives s27 notice to leave in three months’ time.

The tenant serves a s27 notice to leave in the week following the last day of the contractual term.

A

The tenant serves a s27 notice to leave the week before the end of the contractual term.

Correct. The tenant cannot use the section 27 notice to bring the lease to an end before the expiry of the contractual term.

208
Q

From the tenant’s point of view, what is the advantage of a section 26 notice over a section 25 notice?

If the tenant gets the section 26 notice in first, then the landlord loses the opportunity to serve a hostile section 25 notice and therefore oppose renewal.

They are both the same.

A section 26 notice allows the tenant to initiate the lease renewal process if the landlord does not.

If the tenant sets out its desired terms for the draft lease in a section 26 notice, the court is more likely to favour them than the landlord’s proposed counter-terms.

A section 26 notice does not need to include grounds for renewal.

A

A section 26 notice allows the tenant to initiate the lease renewal process if the landlord does not.

Correct. The section 26 notice gives the tenant a mechanism to initiate lease renewal rather than relying on the landlord.

209
Q

When may the parties apply to court to renew the lease?

At any time after service of a section 25 notice, or two months after service of a section 26 notice (or if earlier, the landlord’s service of a counter notice), up until the day before the proposed commencement date of the new lease.

At any time after service of a section 25 notice or section 26 notice, up until the day before the proposed commencement date of the new lease.

At any time after service of a section 25 or 26 notice, up until the day before the proposed commencement date of the new lease. However, for a section 27 notice, the landlord must apply within three months.

No less than 6 months but no more than 12 months following service of a section 25 notice or a section 26 notice.

The time period may be extended indefinitely provided the parties agree over the telephone.

A

At any time after service of a section 25 notice, or two months after service of a section 26 notice (or if earlier, the landlord’s service of a counter notice), up until the day before the proposed commencement date of the new lease.

Correct. This states the correct window for the parties to make an application.

210
Q

A protected lease expired on 5 November. The tenant is holding over, and served a section 26 notice on 10 January following the expiry of the lease. The proposed commencement date is 1 September in the same year as the service of the notice.

As the rent has fallen, the court fixes an interim rent on the tenant’s application. When will this rent date from?

10 January

5 November

1 September

10 April

10 July

A

10 July

Correct. The interim rent will become payable from the earliest date that could be specified in the section 26 notice as the proposed commencement date, ie, 6 months from service of notice.

211
Q

When ordering a renewal lease, which of the following terms can a court not specify?

A lease term of 20 years, where the previous lease was 20 years.

A break clause where the previous lease contained none.

A lease term of 10 years, where the previous lease was 5 years.

A qualified repairing obligation where the previous lease contained a full repairing obligation.

A full repairing obligation where the previous lease contained a qualified repairing obligation.

A

A lease term of 20 years, where the previous lease was 20 years.

Correct. The court can only order a term up to 15 years.

212
Q

A tenant is holding over, and the landlord serves a notice on 31 August this year to bring the lease to an end.

What is the earliest date that can be stated as the proposed termination date?

31 March.

31 August.

31 January.

28 February or 29 February if it is a leap year.

2 March if it is a leap year, or 3 March if it is not.

A

28 February or 29 February if it is a leap year.

Correct. Under the corresponding date rule, as there are not 31 days in February, the latest day of February is taken instead.

213
Q

A tenant (T) under a protected tenancy wants to renew the tenancy and serve a section 26 notice.

The immediate landlord (L1) is itself a tenant of a 10 year lease expiring 3 days after T’s contractual term. L1’s tenancy is not protected.

L1’s landlord (L2) is a 999 year leaseholder, with 925 years left to run.

L2’s landlord (L3) is the freeholder.

Whom should the tenant serve with the section 26 notice?

All of L1, L2 and L3.

On L1 as it is both immediate landlord and a competent landlord.

L3 as it is the competent landlord.

On L2 as it is the first in the chain of landlords who is a competent landlord.

On L1 as it is the immediate landlord.

A

On L2 as it is the first in the chain of landlords who is a competent landlord.

Correct. L2’s interest will not come to an end in 14 months’ time, and therefore L2 qualifies.

214
Q

A protected lease’s contractual term will expire in seven months’ time. The landlord has valid grounds for opposing renewal and both landlord and tenant are certain that they do not wish to continue the tenancy beyond the contractual expiry date, and have told each other this over the phone. Neither party has yet served a notice.

Accepting that notices are mutually exclusive, which statement best summarises the position regarding notices?

No notices are required, as the parties have agreed not to renew.

The landlord may wish to serve a hostile s25 notice.
The tenant may wish to serve a 26 notice.

The landlord may wish to serve a hostile s25 notice.
The tenant may wish to serve a section 27 notice that can bring the tenancy to an end at the end of the contractual term, or may just leave on the last day of the contractual term.

The landlord may wish to serve a hostile s25 notice.
The tenant may wish to serve a s27 notice, but it should not serve a s26 notice as it would then be bound to take a renewal lease.

The landlord may wish to serve a hostile s25 notice.
If the landlord does not, the tenant will need to serve a section 27 notice that brings the tenancy to an end at the end of the contractual term.

A

The landlord may wish to serve a hostile s25 notice.
The tenant may wish to serve a section 27 notice that can bring the tenancy to an end at the end of the contractual term, or may just leave on the last day of the contractual term.